Вы находитесь на странице: 1из 98

www.insightsonindia.

com www.insightsias.com
TEST 4 Solutions

Prelims 2017 Test 4


SOLUTIONS

1. Consider the following about Vinoba Bhave.


1. He undertook a padyatra covering major regions of India to
spread Gandhijis message.
2. He granted the first piece of land to landless villagers; the act
later came to be known as the Bhoodan Movement.
3. Gandhi chose Vinoba to be the first Satyagrahi during the Non-
cooperation movement.

Select the correct answer using the codes below.

a) 1 only

b) 2 and 3 only

c) 1 and 3 only

d) 1 and 2 only

Solution: a)
Justification: Statement 1: Mahatma Gandhi declared Vinoba Bhave
as his spiritual heir. He was one of the votaries of Gandhis concept of
gram swarajya. After Gandhijis martyrdom, Vinobha Bhave undertook
padyatra to spread Gandhijis message covered almost the entire
country.

Statement 2: It was not Vinoba Bhave who granted the land. So, 2 is
wrong.

Once, when he was delivering a lecture at Pochampalli in Andhra


Pradesh, some poor landless villagers demanded some land for
their economic well-being.

Vinoba Bhave could not promise it to them immediately but


assured them to talk to the Government of India regarding
provision of land for them if they undertook cooperative farming.


INSIGHTS PRELIMS TEST SERIES 2017

www.insightsonindia.com www.insightsias.com
TEST 4 Solutions

Suddenly, Shri Ram Chandra Reddy stood up and offered 80 acres


of land to be distributed among 80 landless villagers.

This act was known as Bhoodan.

Statement 3: In 1940 Gandhi chose Vinoba to be the first Satyagrahi i.e.


non-violent resister, to offer non-violent resistance to the British regime.
Individual styagrahis were not choosen during the Non-cooperation
movement. This should be noted very well. So, 3 is wrong.

Q Source: Page 43: Xth Geography NCERT: Contemporary India - II

2. Two nations at Purchasing Power Parity are likely to have


1. Equal currency value in International market
2. Equal Forex reserves
3. Equal national income when measured in a common currency

Select the correct answer using the codes below.

a) 1 and 2 only
b) 3 only
c) 1 and 3 only
d) 2 and 3 only

Solution: b)
Concept: Purchasing power parity is used worldwide to compare the
income levels in different countries. PPP thus makes it easy to
understand and interpret the data of each country.

The PPP theory aims to determine the adjustments needed to be made in


the exchange rates of two currencies to make them at par with the
purchasing power of each other.

In other words, the expenditure on a similar commodity must be same in


both currencies when accounted for exchange rate. The purchasing
power of each currency is determined in the process.


INSIGHTS PRELIMS TEST SERIES 2017

www.insightsonindia.com www.insightsias.com
TEST 4 Solutions

Justification: Statement 3: Suppose sandwich is the only good made


in both India and USA.

It costs Rs. 120 in India and $2 in USA.


Suppose the rupee-dollar exchange rate is 60. If that is the case,
then the market value of sandwich in both nations in the same. You
can buy the same amount of sandwich from 2 dollars as you can
with 120 rupees.
If sandwich was too cheap in India, say Rs. 50, then the two
nations had different purchasing power.
If income levels in both India and USA are same, a US national can
actually buy less sandwiches than an Indian national, as Sandwich
is relatively costlier in USA than India.

Statement 1 and 2: In the above case, even though India and USA are at
purchasing power parity, they need not have equal exchange rates or
equal forex reserves. So, both 1 and 2 are wrong.

Q Source: Improvisation: Page 2: Standard Xth Economics NCERT:


Understanding Economic Development

3. Given below are some sites where major protests were made
against the respective projects. Consider their matches with the
rivers on which they are built.
1. Sardar Sarovar Dam : Narmada
2. Tipaimukh dam: Barak
3. Krishnasagar dam : Krishna
4. Tehri Dam : Mahanadi

The correct matches are?

a) 1 and 4 only
b) 1 and 2 only
c) 2 and 3 only
d) 1, 3 and 4 only


INSIGHTS PRELIMS TEST SERIES 2017

www.insightsonindia.com www.insightsias.com
TEST 4 Solutions

Solution: b)
Justification: Statement 3: Krishnasagar dam was in news due to the
Cauvery dispute between Karnataka and TamilNadu. It is built on
Cauvery. So, 3 is wrong.

Statement 1: The famous Narmada Bachao Andolan was associated with


the same.

Statement 2: It is built on the Barak River in Manipur.

Statement 4: It is on Bhagirathi river. Environmental activist Sunderlal


Bahuguna led the Anti-Tehri Dam movement for years, from 1980s till
2004. The protest was against the displacement of town inhabitants and
environmental consequence of the weak ecosystem.

http://www.firstpost.com/politics/kaveri-river-water-dispute-when-did-
it-start-and-where-are-we-now-2992002.html

http://www.deccanherald.com/content/569917/deve-gowda-visits-
dams-cauvery.html

Q Source: Improvisation Linked to Current Affairs: Page 5: Standard


Xth Economics NCERT: Understanding Economic Development

4. If body weight of a person is given, his Body Mass Index (BMI)


value can reveal his
a) Height
b) Age
c) Nutritional Status
d) Both (a) and (c)

Solution: d)


INSIGHTS PRELIMS TEST SERIES 2017

www.insightsonindia.com www.insightsias.com
TEST 4 Solutions

Justification: One way to find out if adults are undernourished is to


calculate Body Mass Index (BMI).

Take the weight of the person in kg. Then take the height in metres.
Divide the weight by the square of the height. If this figure is less than
18.5 then the person would be considered undernourished. However, if
this BMI is more than 25, then a person is overweight.

Option C: This criterion is not applicable to growing children. So, if we


know the age of the person, he can be calssified in categories of
overweight or underweight.

Option A: By basic mathematics you can derive the height of a person


from his BMI if weight is given.

Q Source: Page 13: Standard Xth Economics NCERT: Understanding


Economic Development

5. Consider the following about National Initiative on Climate


Resilient Agriculture (NICRA).
1. It is a joint project of World Bank and FAO in India.
2. It aims to demonstrate site specific technologies on farmers
fields for adapting to current climate risks.
3. It works towards the capacity building of scientists in climate
resilient agricultural research and its application.

Select the correct answer using the codes below.

a) 1 and 2 only
b) 2 and 3 only
c) 3 only
d) 1, 2 and 3

Solution: b)


INSIGHTS PRELIMS TEST SERIES 2017

www.insightsonindia.com www.insightsias.com
TEST 4 Solutions

Justification: Statement 1: ICAR launched National Initiative on


Climate Resilient Agriculture (NICRA) during 2010-11 for the XIth Plan.
S, 1 is wrong.

Statement 2 and 3: It has following objectives:

To enhance the resilience of Indian agriculture covering crops,


livestock and fisheries to climatic variability and climate change
through development and application of improved production and
risk management technologies

To demonstrate site specific technology packages on farmers fields


for adapting to current climate risks

To enhance the capacity building of scientists and other


stakeholders in climate resilient agricultural research and its
application.

Learning: It follows the following approach to attain the same:

Strengthening the existing net-work research on adaptation and


mitigation (food crops, horticulture, livestock and fishery) with
more infrastructure and capacity building

Setting up of high through put phenotyping platforms and


temperature, CO2, ozone gradient facilities at identified locations/
institutions including North East region.

Strengthening research on climate sensitive crops like cotton,


maize, sugarcane, onion, etc. which are critical for Indias farm
GDP/exports but not covered in the XI Plan

Technologies and Knowledge Resources

Q Source: Improvisation: ICAR Website:


http://www.icar.org.in/en/node/5915: Page 44: Xth Geography NCERT:
Contemporary India - II


INSIGHTS PRELIMS TEST SERIES 2017

www.insightsonindia.com www.insightsias.com
TEST 4 Solutions

6. Kudremukh located in Western Ghats is known for


1. Being one of the global hottest hotspots of biodiversity
2. Being an export dedicated mining unit

Which of the above is/are correct?

a) 1 only
b) 2 only
c) Both 1 and 2
d) None

Solution: c)
Justification: Statement 1: The Kudremukh National Park in the
Western Ghats is a part of the worlds 38 hottest hotspots of biological
diversity, a UNESCO world heritage site.

It forms the largest protected block within the Western Ghats. The park
is home to unique, threatened and endangered biodiversity including
lion-tailed macaques, tigers, Malabar civets and great pied hornbills.

Statement 2: The Kudremukh mines have a unit devoted entirely to


export. The deposits at Kudrcmukh are one of the largest in the world.
They are exploited mostly for export. The ore is transported as slurry
through pipeline to a port near Mangalore.

Q Source: Improvisation: Page 53: Xth Geography NCERT:


Contemporary India - II

7. India is world's third largest importer of crude oil after the United
States and China. Oil imports in India are high and have soared up
in recent years due to which of the following reasons?
a) India does not have sufficient crude oil reserves.
b) Oil Exploration activity in India is sub-optimal and remains much
below potential levels.
c) Domestic crude oil production has been falling in recent years.


INSIGHTS PRELIMS TEST SERIES 2017

www.insightsonindia.com www.insightsias.com
TEST 4 Solutions

d) All of the above

Solution: d)
Learning: Domestic crude oil production fell for the fourth straight
year in 2015-16, even as oil consumption rocketed 11%, pushing up
India's import dependence.

A collapse in oil prices coupled with a rapid economic growth has


helped push up oil consumption at home.
More vehicle purchases, increased use of diesel for irrigation due
to weak monsoon and rising air traffic chiefly drove up
consumption.
As per KPMG officials, exploration activity didn't pick up in the
last decade in India".
The government has unveiled new exploration policies for its oil
and gas blocks lately, aiming to plug loopholes in its previous
policies that encouraged only limited participation of resource-rich
foreign oil companies and couldn't dramatically boost the domestic
output.

Q Source: Improvisation: Page 15: Standard Xth Economics NCERT:


Understanding Economic Development

8. Which of the following is vital for a country to reap its


demographic dividend?
a) Withdrawing food and energy subsidies given to poor citizens
b) Investing in Universal Health Care (UHC) systems
c) Inviting Foreign Institutional Investment (FII)
d) Operating as a Command Economy

Solution: b)


INSIGHTS PRELIMS TEST SERIES 2017

www.insightsonindia.com www.insightsias.com
TEST 4 Solutions

Justification: One of Indias competitive advantages is its demographic


dividend. Demographic dividend occurs when the proportion of working
people in the total population is high because this indicates that more
people have the potential to be productive and contribute to growth of
the economy.

Option A: Withdrawing such critical support systems may make the poor
worse off and likely to lead to greater impoverishment than enhanced
demographic dividend.

Option B: UHC systems can be crucial to building a healthy workforce


and augment human capital. So, B is correct.

Option C: FII is short-term in nature and doesnt contribute to


permanent capital formation in the nation. So, C is wrong.

Option D: A command economy is characterized by state control and


state-led planning. Many East Asian nations have been able to reap
demographic dividend without resorting to such a strict control system.
So, D is wrong.

Q Source: Improvisation: Chapter 1: Standard Xth Economics NCERT:


Understanding Economic Development

9. How are highways numbered in India as per the system introduced


by the Ministry of Road Transport and Highways in 2010?
1. National highways from the East to West have odd numbers
while those from the North to South are even numbered.
2. Major highways are enumerated in single or double-digit
numbers.

Which of the above is/are correct?

a) 1 only
b) 2 only


INSIGHTS PRELIMS TEST SERIES 2017

www.insightsonindia.com www.insightsias.com
TEST 4 Solutions

c) Both 1 and 2
d) None

Solution: c)
Justification & Learning: According to the new rationalized
numbering (introduced since 2010:

All East-West highways now have odd numbers, starting in the


North and increasing in number towards the South.

All North-South highways now have even numbers, starting in the


East and increasing in number towards the West. I

Major highways have single- or double-digit numbers.

Suffixes A, B, C, D etc are added to the three digit sub highways to


indicate very small spin-offs or stretches of sub-highways.

For e.g. the new NH-2 is the highway from Dibrugarh to Sibsagar
and Amguri to Mokochong, Wokha and Kohima in Nagaland to
Imphal, Churachandpur in Manipur to Seling, Serchhip and
Tuipang in Mizoram.

Q Source: Improvisation: Page 82: Xth Geography NCERT:


Contemporary India - II

10. Consider the following about Hydraulic Structures in Ancient


India.
1. The Mauryan regulated and taxed water structures used for
irrigating crop fields.
2. Sringaverapura, first Century B.C, hosted sophisticated water
harvesting systems for channelling flood water of the Ganges.
3. Iltutmish is known for dismantling the major hydraulic
structures of Delhi as a part of his war strategy.

Select the correct answer using the codes below.


INSIGHTS PRELIMS TEST SERIES 2017

www.insightsonindia.com www.insightsias.com
TEST 4 Solutions

a) 1 and 2 only
b) 2 and 3 only
c) 1 only
d) 1 and 3 only

Solution: a)
Justification: Statement 1: At Sringaverapura near Allahabad in Uttar
Pradesh, India, there exists an extraordinary example of hydraulic
engineering dating back to the end of the 1st century BC. It comprises
three percolation-cum-storage tanks, fed by a deep canal that used to
skim the floodwaters off the Ganga.

Statement 2: The Arthashastra of Kautilya gives an extensive account of


dams and bunds that were built for irrigation during the period of the
Mauryan Empire.

The water supply systems were well managed within the


framework of strict rules and regulations. Different types of taxes
were collected from the cultivators depending upon the nature of
irrigation.

Statement 3: The opposite is true. In the 14th Century, the tank in Hauz
Khas, Delhi was constructed by Iltutmish for supplying water to Siri Fort
area.

Also, in the 11th Century, Bhopal Lake, one of the largest artificial lakes
of its time was built.

Q Source: Improvisation: Page 26: Xth Geography NCERT:


Contemporary India - II

11. Indian pangolins are under threat due to large scale hunting and
poaching for consumptive uses of
1. Protein source
2. Traditional Medicine
3. Agar producing Jelly


INSIGHTS PRELIMS TEST SERIES 2017

www.insightsonindia.com www.insightsias.com
TEST 4 Solutions

Select the correct answer using the codes below.

a) 1 and 2 only
b) 2 and 3 only
c) 1 and 3 only
d) 1, 2 and 3

Solution: a)
Justification: Pangolins are the most heavily trafficked CITES-
protected mammal.

It is estimated that 100,000 pangolins are captured every year from


across Africa and Asia, with most shipped to China and Vietnam, where
their meat and scales are sold.

As a result, all eight species of pangolin now feature on the International


Union for the Conservation of Nature (IUCN) Red List of animals.

Pangolin meat is a rich source of protein. Its scales are sold by the bag in
Asia, where some believe they can cure everything from cancer to acne.

Q Source:
http://www.wwfindia.org/about_wwf/priority_species/threatened_spe
cies/indian_pangolin/

12.Disguised unemployment will result if


a) A firm hires too many workers to accomplish a one-man job
b) A large number of people in a nation are engaged in the services
sector
c) The output from industrial sector is low
d) There is high unemployment and heavy migration

Solution: a)
Justification & Learning: For e.g. there are more people in Indian
agriculture than is necessary. So, even if you move a few people out,


INSIGHTS PRELIMS TEST SERIES 2017

www.insightsonindia.com www.insightsias.com
TEST 4 Solutions

production will not be affected. In other words, workers in agricultural


sector are under-employed.

Each one is doing some work but no one is fully employed.

This kind of underemployment is hidden in contrast to someone who


does not have a job and is clearly visible as unemployed. Hence, it is also
called disguised unemployment.

Q Source: Page 26: Standard Xth Economics NCERT: Understanding


Economic Development

13. Consider the following about the Mahatma Gandhi National Rural
Employment Guarantee Act, 2005 (MGNREGA).
1. The Village Panchayat issues job cards to every registered
individual.
2. Rural construction projects are generally handed over to private
contractors who are made responsible to the Gram Panchayat.
3. The cost of the scheme is shared between the Centre and the
States.

Select the correct answer using the codes below.

a) 1 and 2 only
b) 2 only
c) 1 and 3 only
d) 1, 2 and 3

Solution: c)
Justification: The Mahatma Gandhi National Rural Employment
Guarantee Act, 2005 (MGNREGA) guarantees 100 days of employment
in a financial year to any rural household whose adult members are
willing to do unskilled manual work.

Statement 2: Use of contractors in commissioning works is strictly


banned under the Act.


INSIGHTS PRELIMS TEST SERIES 2017

www.insightsonindia.com www.insightsias.com
TEST 4 Solutions

Learning: The objective of the Act is to create durable assets and


strengthen the livelihood resource base of the rural poor.

The choice of works suggested in the Act address causes of chronic


poverty like drought, deforestation, soil erosion, so that the process
of employment generation is on a sustainable basis.
Works suggested in the Act addresses causes of chronic poverty
like drought, deforestation and soil erosion, so that the process of
employment generation is maintained on a sustainable basis.

Q Source: Improvisation: Chapter 2: Standard Xth Economics NCERT:


Understanding Economic Development

14.The Union Cabinet has recently approved the process, formation


and functioning of the Goods and Services Tax (GST) Council.
Consider the following about it.
1. It will be a constitutional body.
2. It will be a joint forum of the Centre and the States chaired by
the Prime Minister.
3. The Council will make recommendations to the Union and the
States on GST rates.

Select the correct answer using the codes below.

a) 1 and 2 only
b) 2 and 3 only
c) 1 and 3 only
d) 3 only

Solution: c)
Justification: As per Article 279A of the amended Constitution, GST
Council which will be a joint forum of the Centre and the States. It shall
have the following composition:


INSIGHTS PRELIMS TEST SERIES 2017

www.insightsonindia.com www.insightsias.com
TEST 4 Solutions

Union Finance Minister (Chairperson).

The Union Minister of State (MoS) in-charge of Revenue of finance


(Member)

The Minister In-charge of taxation or finance or any other Minister


nominated by each State Government (Members)

Functions of GST Council

As per Article 279A (4), the Council will make recommendations to


the Union and the States on important issues related to GST, like

Goods and services that may be subjected or exempted from GST.

Model GST Laws

Principles that govern Place of Supply, threshold limits

GST rates including the floor rates with bands, special rates for
raising additional resources during natural disasters/ calamities,
special provisions for certain States, etc.

Q Source: Approval of GST council establishment by Cabinet

15. In one of the biggest initiatives towards institution building,


NABARD has facilitated the process of bringing the Co-operative
Sector onto Core Banking Solution (CBS) platform. How will CBS
be helpful to the sector?
1. It will allow seamless merger and acquisition of banks in case of
regulatory and financial failures.
2. It will bring transparency and correction of books of account of
these banks to arrest pilferage of funds.

Which of the above is/are correct?

a) 1 only
b) 2 only


INSIGHTS PRELIMS TEST SERIES 2017

www.insightsonindia.com www.insightsias.com
TEST 4 Solutions

c) Both 1 and 2
d) None

Solution: b)
Justification: Bringing the Co-operative Banks under Core banking
Solution would provide the following advantages

Bring them at par with the technological platform of the


Commercial Banks and also RRB and match up with these
institutions in providing similar kind of services in the
hinterland of the country.
Transparency and regular reconciliation and balancing of books
of account to arrest pilferage.
Once the Banks are on a technology platform they will be in a
position to utilise technological solution furthering the financial
inclusion initiative in rural India
The customer of the Banks can avail facilities like Any Branch
Banking, e-transfer of funds to anywhere of the country, etc.
This will also enable GOI to transfer incentive/subsidy/other
payments to the account holders based on Aadhaar number.
Technological solution is expected to release staff members so
that there will be more follow-up with the customers and also
help bring in new customers/ clients to the Banks thus
improving the business portfolio.

Q Source: Improvisation: Chapter 3 mention of NABARD: Standard


Xth Economics NCERT: Understanding Economic Development

16.Lichens are a symbiotic relationship between


1. Fungi
2. Bacteria
3. Algae
4. Virus


INSIGHTS PRELIMS TEST SERIES 2017

www.insightsonindia.com www.insightsias.com
TEST 4 Solutions

Select the correct answer using the codes below.

a) 1 and 2 only
b) 1 and 3 only
c) 2 and 4 only
d) 3 and 4 only

Solution: b)
Justification: A lichen is not a single organism. Rather, it is a
symbiosis between different organisms - a fungus and an alga or
cyanobacterium.

The lichen fungi (kingdom Fungi) cultivate partners that


manufacture food by photosynthesis. Sometimes the partners are
algae (kingdom Protista), other times cyanobacteria (kingdom
Monera), formerly called blue-green algae.

Lichens can be found growing in almost all parts of the terrestrial


world, from the ice-free polar areas to the tropics, from tropical
rainforests to those desert areas free of mobile sand dunes. While
generally terrestrial a few aquatic lichens are known.

Learning: Most lichens grow very, very slowly, often less than a
millimeter per year, and some lichens are thought to be among the oldest
living things on Earth.

Lichens with known, slow growth rates, like Rhizocarpon geographicum,


have been used to estimate the dates of geological events such as the
retreat of glaciers.

Q Source: Page 84: IXth Science NCERT

17. Delhi first became the capital of a kingdom under the


a) Tomara Rajputs
b) Chauhans
c) Dehliwals


INSIGHTS PRELIMS TEST SERIES 2017

www.insightsonindia.com www.insightsias.com
TEST 4 Solutions

d) Tuglaq

Solution: a)
Learning: Tomara Rajputs were later defeated in the middle of the
twelfth century by the Chauhans (also referred to as Chahamanas) of
Ajmer.

It was under the Tomaras and Chauhans that Delhi became an important
commercial centre.

Coins minted here, called dehliwal, had a wide circulation.

The transformation of Delhi into a capital that controlled vast areas of


the subcontinent started with the foundation of the Delhi Sultanate in
the beginning of the thirteenth century.

Q Source: Revision past test syllabus: Improvisation: Page 30: Class 7th
NCERT History: Our Pasts-II

18. Gram Gadar (Village Revolution), a wall newspaper is


published by
a) The Council for Advancement of People's Action and Rural
Technology (CAPART)
b) Goonj NGO
c) Centre for Social Research:
d) None of the above

Solution: d)
Learning: CUTS International (Consumer Unity & Trust Society) began
its journey in 1983 in Rajasthan, from a rural development
communication initiative, a wall newspaper Gram Gadar (Village
Revolution).


INSIGHTS PRELIMS TEST SERIES 2017

www.insightsonindia.com www.insightsias.com
TEST 4 Solutions

This monthly wall newspaper is published regularly and has been


instrumental in providing a forum for the oppressed classes to get
justice.

On seeing Gram Gadar, Rubens Ricupero, Secretary-General of


UNCTAD (1995-2004) observed: It confirmed my view that often the
simple lack of awareness lies at the root of so much misery.

Q Source: Improvisation: Page 74 & 78: Standard Xth Economics


NCERT: Understanding Economic Development

19.In one of the northeastern states of India, an ingenious system of


tapping of stream and springwater by using bamboo pipes to
irrigate plantations is widely prevalent. The state is?
a) Arunachal Pradesh
b) Meghalaya
c) Manipur
d) Assam

Solution: b)
Learning: The 200 year old system is found in the war areas of
Meghalaya but is more prevalent in the war Jaintia hills than in the
war Khasi hills.

The region has very steep slopes and a rocky terrain. Diverting water
through ground channels is not possible.

This timeless and traditional technology uses locally available


material while harnessing the forces of gravity.

The land used for cultivation is owned by the clan, and is allocated
for cultivation by the clan elders on payment of a one-time rent.
The clan elders have the prerogative to decide who should get what
and how much land.


INSIGHTS PRELIMS TEST SERIES 2017

www.insightsonindia.com www.insightsias.com
TEST 4 Solutions

Q Source: Page 32: Xth Geography NCERT: Contemporary India - II

20. Consider the following about the Annual Public Forum,


WTOs flagship event.


INSIGHTS PRELIMS TEST SERIES 2017

www.insightsonindia.com www.insightsias.com
TEST 4 Solutions

1. It is the publisher of the World Trade Statistical Review since


1995.
2. It provides a common platform for heads of state as well as
NGOs to discuss major trade and developmental issues.

Which of the above is/are correct?

a) 1 only
b) 2 only
c) Both 1 and 2
d) None

Solution: b)
Justification: Statement 1: The WTO has launched a new annual
statistical publication the World Trade Statistical Review in 2016.
This new publication provides insights into how world trade has evolved
in recent years by analysing the latest trade statistics within an economic
context. So, 1 is wrong.

Statement 2: It provides a unique platform for heads of state and


leading global businesspeople, academics and non-governmental
organisations to come together and discuss some of the major trade and
development issues of the day. Over 1,500 participants attend the Forum
each year.

Q Source: WTO website http://www.wto.org: Improvisation: Page 54:


Standard Xth Economics NCERT: Understanding Economic
Development

21.Which of the following locations is nearest to the Equator?


a) Male
b) Jakarta
c) Bandung
d) Yogyakarta


INSIGHTS PRELIMS TEST SERIES 2017

www.insightsonindia.com www.insightsias.com
TEST 4 Solutions

Solution: a)
Justification: Male has the following lat-long - 4.1755 N, 73.5093 E;
Jakarta has the following - 6.1745 S, 106.8227 E.

Bandung and Yogyakarta lie further South of Jakarta. So, Male lies
closest to the equator among the given options.

Q Source: South-east Asia and South Asia: Map-based questions

22. Consider the following statements about a bird species.


1. It is listed as Vulnerable in the IUCN redlist of threatened
species.
2. It is endemic to Central India.
3. Illegal trade is the biggest threat to this species due to which it
has been covered under CITES.
4. It is a highly sought after pet due to its melodious call.

The above refer to?

a) Koyna
b) Common Myna
c) Black-necked Crane
d) Green Munia


INSIGHTS PRELIMS TEST SERIES 2017

www.insightsonindia.com www.insightsias.com
TEST 4 Solutions

Solution: d)
Justification: India is home to eight species of Munias, belonging to
the family Estrildidae.

These small birds, found in different parts of India, are brightly


coloured and have melodious calls, making them highly sought
after cage birds, and victims of the pet trade in India.

Its population is unevenly distributed across southern Rajasthan,


central Uttar Pradesh, Madhya Pradesh, southern Bihar and West
Bengal, southern Maharashtra and Northern Andhra Pradesh.

Munias are listed in Schedule IV of the Wildlife (Protection) Act,


1972, making hunting, trapping or trade illegal and a punishable
offence.

Q Source:
http://www.wwfindia.org/about_wwf/priority_species/lesser_known_s
pecies/green_munia/

23. When you buy a medicine, you will find certain details given
on the packing which include
1. Ingredients used
2. Address of the manufacturer


INSIGHTS PRELIMS TEST SERIES 2017

www.insightsonindia.com www.insightsias.com
TEST 4 Solutions

3. Directions for proper use


4. Name of the brand endorser
5. Information relating to side effects and risks associated with the
same

Select the correct answer using the codes below.

a) 1, 2 and 3 only
b) 1, 2, 3 and 5 only
c) 4 and 5 only
d) 1, 2, 3, 4 and 5

Solution: b)
Justification: These details are about ingredients used, price, batch
number, date of manufacture, expiry date and the address of the
manufacturer.

When we buy medicines, on the packets, you might find directions for
proper use and information relating to side effects and risks associated
with usage of that medicine.

When you buy garments, you will find information on instructions for
washing.

Consumers have the right to be informed about the particulars of goods


and services that they purchase.

Consumers can then complain and ask for compensation or replacement


if the product proves to be defective in any manner.

Q Source: Page 80: Standard Xth Economics NCERT: Understanding


Economic Development

24. Strait of Malacca is a waterway


1. Connecting the Andaman Sea in the Indian Ocean and the
South China Sea


INSIGHTS PRELIMS TEST SERIES 2017

www.insightsonindia.com www.insightsias.com
TEST 4 Solutions

2. Running between the Malay Peninsula and Sumatran Island in


Indonesia

Which of the above is/are correct?

a) 1 only
b) 2 only
c) Both 1 and 2
d) None

Solution: c)
Justification: It is a narrow, 850 km stretch of water between the
Malay Peninsula and the Indonesian island of Sumatra.

It is a strategic choke point for shipping supplies from East Asia heading
to South Asia. The strait is the main shipping channel between the
Indian Ocean and the Pacific Ocean, linking major Asian economies such
as India, China, Japan, Taiwan, and South Korea.

About a quarter of all oil carried by sea passes through the Strait, mainly
from Persian Gulf suppliers to Asian markets

Q Source: South-east Asia: Map-based questions

25. The Great Nicobar Biosphere Reserve is known to host which


of the following important species?
1. Dugong
2. Salt Water crocodile
3. Great Indian Hornbill

Select the correct answer using the codes below.

a) 1 and 2 only
b) 2 only
c) 3 only
d) 1 and 3 only


INSIGHTS PRELIMS TEST SERIES 2017

www.insightsonindia.com www.insightsias.com
TEST 4 Solutions

Solution: a)
Justification: The Great Hornbill is the State bird of Chin state in
Myanmar, and Kerala and Arunachal in India. It is not found in A&N
Islands. It is Narcondam hornbill that is found in A&N. So, 3 is incorrect.

Dugong is found in India in Gulf of Mannar, A&N and Gulf of Kutch.

Salt water crocodile is found in A&N islands and the eastern coasts of
India.

Q Source: Biosphere Reserves of India in the UNESCO Man and


Biosphere Network

26. Which of the following decides the Issue Price of Food Grains
in India?
a) Cabinet Committee on Economic Affairs (CCEA)
b) Food Corporation of India (FCI)
c) Commission on Agricultural Costs and Prices (CACP)
d) Ministry of Food, Public Distribution and Consumer Affairs

Solution: a)
Justification: A buffer stock is created by the government to distribute
food grains in the deficit areas and among the poorer strata of society at
a price lower than the market price also known as Issue Price.

FCI procures the food grains. It does not decide the issue price. So, B is
wrong.

CACP only recommends prices to the CCEA. It is the CCEA which finally
decides the issue price. CACP is not authorized to fix them. So, C is
wrong.

Learning: FCI plays a pivotal role in grain distribution and ensuring


food security in India.


INSIGHTS PRELIMS TEST SERIES 2017

www.insightsonindia.com www.insightsias.com
TEST 4 Solutions

The FCI purchases wheat and rice from the farmers in states where
there is surplus production.
The farmers are paid a pre-announced price for their crops. This
price is called Minimum Support Price.
The MSP is declared by the government every year before the
sowing season to provide incentives to the farmers for raising the
production of these crops.

Q Source: Page 47: Standard IXth Economics NCERT

27. All important rivers and streams of the Garo Hills region rise
from the Mountain Range located in the Protected Area. The
protected area also hosts the Nongkhyllem Wildlife Sanctuary and
Balpakram National Park other than hosting the Hollock Gibbon,
the only ape species of India. It is?
a) Nokrek National Park
b) Namdapha Reserve
c) Dachigham Reserve
d) Manas Biosphere Reserve

Solution: a)
Justification: The entire Biosphere Reserve is hilly. Important rivers
and streams of the Garo Hills region rise from the Nokrek Range, of
which the river Simsang, known as Someshwari when it emerges into
Bangladesh at Baghmara, is the most prominent.

Balpakram National Park and the famous Siju Cave is located very
close to the Napak Lake near the Simsang River game reserve
which is south east to Nokrek National Park.

The Garos never kill or hunt the Hoolock Gibbon as it is a


traditional belief among them that if a Hoolock Gibbon is killed, a
famine or a curse would befall the entire village.


INSIGHTS PRELIMS TEST SERIES 2017

www.insightsonindia.com www.insightsias.com
TEST 4 Solutions

UNESCO added this National park to its list of Biosphere Reserves


in 2009. Along with Balphakram national park, Nokrek is a
hotspot of biodiversity in Meghalaya

Q Source: Biosphere Reserves of India in the UNESCO Man and


Biosphere Network

28. National Food Security Act allocates subsidized food grains


to
a) All Indian citizens
b) Only Below Poverty Line citizens
c) All agricultural households
d) A certain fixed percentage of urban and rural population

Solution: d)
Learning: This Act provides for food and nutritional security life at
affordable prices and enable people to live a life with dignity.

Under this act 75% of rural population and 50% of urban population
have been categorised as eligible households for food security.

It also includes provisions for modernization of PDS system, end to end


computerization of the supply chain, one time hot meal to pregnant
women and other measures to improve stockholding of grains to reduce
corruption and pilferage.

Q Source: Page 49: Standard IXth Economics NCERT

29. Consider the following statements about plant pathogens.


INSIGHTS PRELIMS TEST SERIES 2017

www.insightsonindia.com www.insightsias.com
TEST 4 Solutions

1. Bacteria as well as fungi can cause diseases in plants.


2. Pathogens can affect plants only after transmitting itself from
the root to the shoots.
3. A pathogen cannot be transmitted to plant by the medium of
water.

Select the correct answer using the codes below.

a) 1 only
b) 2 and 3 only
c) 3 only
d) 1, 2 and 3

Solution: a)
Justification: The following diagram addresses the answer.

Pathogens can transmit themselves through all three modes air, water
and soil.

Q Source: Revision questions: Previous Tests


INSIGHTS PRELIMS TEST SERIES 2017

www.insightsonindia.com www.insightsias.com
TEST 4 Solutions

30. Which of the following statements is INCORRECT about


National Council of Senior Citizens which is chaired by the
Minister of Social Justice & Empowerment?
a) The council was originally known as National Council for Older
Persons (NPOP).
b) The Council is a permanent body that meets on a daily basis.
c) It is an advisory body to both Central and State governments.
d) None of the above

Solution: b)
Justification: You can solve this question by elimination.

If the body is chaired by a person like the Minister of Social Justice &
Empowerment, it is NOT possible for the body to meet on a daily basis
given the busy schedule of the Ministers. The body meets twice an year.
So, B is incorrect.

Not so important facts like this are often asked by UPSC. You should also
pay attention to these details.

Learning: The mandate of the National Council of Senior Citizens is to


advise the Central and State Governments on the entire gamut of issues
related to the welfare of senior citizens and enhancement of their quality
of life, with special reference to the following:-

Policies, programmes and legislative measures.

Promotion of physical and financial security, health and


independent and productive living.

Awareness generation and community mobilization.

Q Source: http://www.insightsonindia.com/2016/08/31/insights-
daily-current-affairs-31-august-2016/


INSIGHTS PRELIMS TEST SERIES 2017

www.insightsonindia.com www.insightsias.com
TEST 4 Solutions

31. Consider the following about Earth Summit.


1. It was convened for the first time in 1972 after the publication of
the Limits to Growth Thesis.
2. It is convened each year to address urgent environmental
protection and socio-economic development at the global level.

Which of the above is/are correct?

a) 1 only
b) 2 only
c) Both 1 and 2
d) None

Solution: d)
Justification: Statement 2: In 1992, more than 100 heads of states met
in Rio de Janeiro in Brazil, for the first International Earth Summit. The
Summit was convened for addressing urgent problems of environmental
protection and socioeconomic development at the global level. So, 1 is
wrong.

The assembled leaders signed the Declaration on Global Climatic Change


and Biological Diversity.

Statement 1: Earth Summit 2012 was held 20 years after the 1992
Summit. So, 2 is wrong.

Q Source: Page 3: Xth Geography NCERT: Contemporary India - II

32. The Residents appointed by the East India Company looked


after
a) Provincial revenue collection
b) Administration of Madrassas and Missionaries


INSIGHTS PRELIMS TEST SERIES 2017

www.insightsonindia.com www.insightsias.com
TEST 4 Solutions

c) Implementation of the land tenure system


d) Political and commercial affairs of the princely states

Solution: d)
Learning: After the Battle of Buxar (1764), the Company appointed
Residents in Indian states.

They were political or commercial agents and their job was to serve and
further the interests of the Company.

Through the Residents, the Company officials began interfering in the


internal affairs of Indian states.

They tried to decide who was to be the successor to the throne, and who
was to be appointed in administrative posts.

Q Source: Revision previous Test syllabus

33. If you draw a line between Vladivostok and Macau on a map,


which of these will lie on the right hand side of the line?
1. Spartly Islands
2. Paracel Islands
3. Phillipines Sea

Select the correct answer using the codes below.

a) 1 only
b) 1 and 2 only
c) 2 and 3 only
d) 1, 2 and 3

Solution: d)
Justification: Vladivostok is one of the easternmost points of Russia.
Macau is located near Hong Kong.


INSIGHTS PRELIMS TEST SERIES 2017

www.insightsonindia.com www.insightsias.com
TEST 4 Solutions

Use elimination here. Islands (Spartly and Paracel) are disputed islands
between China and other South-east Asian nations.

Phillipines Sea lies to the East to these islands. So, if an option contains
these islands, it must also contain the Sea. In this case, the only answer
possible can be option D.

Learning: Map is below.

Q Source: East Asia: Map-based questions

34. In which of the following states of India, the Net Sown Area
as a percentage of total area is highest?
a) Arunachal Pradesh
b) Mizoram
c) Punjab


INSIGHTS PRELIMS TEST SERIES 2017

www.insightsonindia.com www.insightsias.com
TEST 4 Solutions

d) Manipur

Solution: c)
Justification: All areas except Option C are largely hilly and heavily
forested areas, which are not suitable for agriculture.

Net Sown Area is over 80 per cent of the total area in Punjab and
Haryana.

It is less than 10 per cent of total areas in Arunachal Pradesh, Mizoram,


Manipur and Andaman Nicobar Islands.

Q Source: Page 6: Xth Geography NCERT: Contemporary India - II

35. It is the State animal of Manipur and found in Keibul Lamjao


National Park. It is largely seen over the floating biomass, locally
called phumdi. The animal is
a) Sangai
b) Wild Buffalo
c) Nil Makru
d) Golden Mahseer

Solution: a)
Learning: The brow-antlered deer (Sangai) is found in the South
Eastern part of Loktak Lake inside the park.

It is covered under Schedule-1 of Wildlife (Protection) Act, 1972


and Endangered on IUCN Red List.

The deer walks on the hind surface of its pasterns with mincing
hops over floating foliage, and is hence also called the Dancing
Deer.

Phumdi is the most important and unique part of Sangais habitat.


INSIGHTS PRELIMS TEST SERIES 2017

www.insightsonindia.com www.insightsias.com
TEST 4 Solutions

It is the floating mass of entangled vegetation formed by the


accumulation of organic debris and biomass with soil.

It floats with 4/5 part under water.

Q Source:
http://www.wwfindia.org/about_wwf/priority_species/threatened_spe
cies/brow_antlered_deer/

36. Consider the following about the harmful effects of mining


and mineral processing.
1. Deforestation caused due to mining has caused severe land
degradation in major Eastern Coal belts of India.
2. The dust generated by limestone processing Industry clogs soil
pore adversely affecting water infiltration in the soil.

Which of the above is/are correct?

a) 1 only
b) 2 only
c) Both 1 and 2
d) None

Solution: c)
Justification: Statement 1: Mining sites are abandoned after
excavation work is complete leaving deep scars and traces of over-
burdening.

In states like Jharkhand, Chhattisgarh, Madhya Pradesh and Orissa


deforestation due to mining have caused severe land degradation.

Statement 2: The mineral processing like grinding of limestone for


cement industry and calcite and soapstone for ceramic industry generate
huge quantity of dust in the atmosphere.


INSIGHTS PRELIMS TEST SERIES 2017

www.insightsonindia.com www.insightsias.com
TEST 4 Solutions

It retards the process of infiltration of water into the soil after it settles
down on the land. In recent years, industrial effluents as waste have
become a major source of land and water pollution in many parts of the
country.

Q Source: Page 7: Xth Geography NCERT: Contemporary India - II

37. Arrange these physical features of Asia from North to South.


1. Kunlun Shan Mountains
2. Gobi Desert
3. Arakan mountains

Select the correct answer using the codes below.

a) 231
b) 132
c) 123
d) 213

Solution: d)
Learning: The Arakan Mountains is located in western Myanmar,
between the coast of Rakhine State and the Central Burma Basin, in
which flows the Irrawaddy River.

Since Myanmar lies further south of the statements 1 and 2, only option
C or D can be the answer. If you know the location of Kunlun shan
mountains, the answer can be easily market as D.


INSIGHTS PRELIMS TEST SERIES 2017

www.insightsonindia.com www.insightsias.com
TEST 4 Solutions

Q Source: East Asia: Map-based questions

38. The Reserve Bank of India supervises the functioning of


banks in which of the following ways?
1. RBI monitors the banks so that they maintain prescribed cash
balance
2. RBI by its mandate prohibits banks from earning excessive
profits
3. RBI supervises the credit activities of lenders in the informal
sector
4. RBI ensures that bank loans are diverted towards priority
sectors to achieve inclusive growth

Select the correct answer using the codes below.

a) 2, 3 and 4 only
b) 2 and 3 only
c) 1 and 4 only
d) 1, 2, 3 and 4


INSIGHTS PRELIMS TEST SERIES 2017

www.insightsonindia.com www.insightsias.com
TEST 4 Solutions

Solution: c)
Justification: Statement 1: Banks maintain a minimum cash balance
out of the deposits they receive. The RBI monitors the banks in actually
maintaining cash balance.

Statement 4: Similarly the RBI sees that the banks give loans not just to
profit-making businesses and traders but also to small cultivators, small
scale industries, to small borrowers etc.

Periodically, banks have to submit information to the RBI on how much


they are lending, to whom, at what interest rate, etc.

Statement 3: There is no organisation which supervises the credit


activities of lenders in the informal sector. They lend at arbitrary interest
rates.

Q Source: Improvisation: Page 49: Standard Xth Economics NCERT:


Understanding Economic Development

39. Consider the following about Apatani tribes.


1. They are one of the major ethnic groups of eastern Himalayas.
2. The tribe is known for its vibrant traditional village councils
called bulyan.
3. UNESCO recognizes the wet fish-rice cultivation technique of
Apatani tribes as highly efficient.

Select the correct answer using the codes below.

a) 1 and 2 only
b) 3 only
c) 1 only
d) 1, 2 and 3

Solution: d)


INSIGHTS PRELIMS TEST SERIES 2017

www.insightsonindia.com www.insightsias.com
TEST 4 Solutions

Justification: Statement 1: The Ziro valley is inhabited by the Apatani


tribe in the lower ranges of the eastern Himalayas in the state of
Arunachal Pradesh.

Statement 2: UNESCO notes, The Apatanis, one of the major ethnic


groups of eastern Himalayas, have a distinct civilization with systematic
land use practices and rich traditional ecological knowledge of natural
resources management and conservation, acquired over the centuries
through informal experimentation.

The tribe is known for their colorful culture with various festivals,
intricate handloom designs, skills in cane and bamboo crafts, and
vibrant traditional village councils called bulya.

Statement 3: It further notes, The community has evolved a unique skill


of rice-fish cultivation where along with paddy, fish is also reared on the
fields. This is further supplemented with millet (Eleusine coracana)
reared on elevated partition bunds between the rice plots.

In spite of limited water resources the entire expanse of the cultivated


area in the valley is well watered by a network of meticulously
engineered irrigation channels.

Such an ingenious traditional system sets valuable example, especially in


the face of impending global warming and threats of water scarcity all
over the world.

Q Source: Novel based on Apatani tribe of Arunachal Pradesh titled


Into the Hidden Valley has won 2016 MM Bennetts Award for Historical
Fiction in the United Kingdom.

40. Which of the following are categorized by IUCN as


Vulnerable/ Endangered/Near Threatened species as notified by
IUCN?
a) Black buck, Crocodile and Indian wild ass
b) Lion tailed macaque and Blue sheep


INSIGHTS PRELIMS TEST SERIES 2017

www.insightsonindia.com www.insightsias.com
TEST 4 Solutions

c) Himalayan brown bear and Wild Asiatic buffalo


d) Desert fox and hornbill

Solution: a)
Learning: Blue sheep is a species of Least Concern as per IUCN. So, B
is wrong.

Desert fox and hornbill are rare species. So, C is wrong.

Black Buck and Indian Wild Ass are near threatened as per IUCN.
Crocodile is Vulnerable.

It is possible that other sources show a different conservation status of


these species, as the database may not be updated at other sources.

Q Source: Page 15: Xth Geography NCERT: Contemporary India - II

41.Consider the following matches of tiger reserves with the state they
are located in.
1. Corbett National Park : Uttar Pradesh
2. Bandhavgarh National Park : Odisha
3. Periyar Tiger Reserve : Kerala

Select the correct answer using the codes below.

a) 1 and 2 only
b) 3 only
c) 2 only
d) None of the above

Solution: b)
Learning: Some of the important national parks of India that are also
tiger reserves with their locations:


INSIGHTS PRELIMS TEST SERIES 2017

www.insightsonindia.com www.insightsias.com
TEST 4 Solutions

Corbett located in Uttarakhand. Sunderbans National Park in West


Bengal, Bandhavgarh National Park in Madhya Pradesh, Sariska Wildlife
Sanctuary in Rajasthan, Manas Tiger Reserve in Assam and Periyar Tiger
Reserve in Kerala are some of the tiger reserves of India.

Q Source: Page 19: Xth Geography NCERT: Contemporary India - II

42. Gharial population in India is declining and at threat due to


1. Getting entangled in fishing nets
2. Construction of dams and barrages
3. Egg harvesting for subsistence food use
4. Removal of sand from its habitat riverbanks

Select the correct answer using the codes below.

a) 1, 2 and 3 only
b) 1 and 4 only
c) 2 and 3 only
d) 1, 2, 3 and 4

Solution: d)
Justification: Statement 1: Gharial, with its long, toothy rostrum is
particularly vulnerable to entanglement in fishing nets, where it is
frequently trapped underwater and drowns.

Entangled gharial are also commonly killed or have their rostrums


chopped off to disentangle nets and perhaps, in retaliation for damaging
nets.

Statement 2: Dam, barrages, and water abstraction adversely affects


gharial by turning suitable river habitats into marginal/ unsuitable lakes,
and by altering the quantity and quality of water available to
downstream river sections.

Statement 3: Egg harvesting for subsistence food use by riparian


residents at some gharial locations directly increases egg mortality,


INSIGHTS PRELIMS TEST SERIES 2017

www.insightsonindia.com www.insightsias.com
TEST 4 Solutions

reduces recruitment, and may also facilitate additional predation by


natural nest predators.

Statement 4: Sustained mining activity may destroy vital basking and


nesting sites and may also result in direct mortality of eggs during the
nesting season.

Learning: Gharial derives its name from ghara, an Indian word for pot
because of a bulbous knob present at the end of their snout.

Q Source: Improvisation: Page 20: Xth Geography NCERT:


Contemporary India - II

43. Consider the following statements.


1. All activities are permitted unless restricted in protected forests,
whereas in reserved forests all activities are restricted unless
permitted.
2. Reserved forests and protected forests both can be notified by
the State governments.

Which of the above is/are correct?

a) 1 only
b) 2 only
c) Both 1 and 2
d) None

Solution: b)
Justification: Statement 1: Rights to all activities like hunting, grazing,
etc. in reserved forests are banned unless specific orders are issued
otherwise. In protected areas, rights to activities like hunting and grazing
are sometimes given to communities living on the fringes of the forest,
who sustain their livelihood partially or wholly from forest resources or
products.


INSIGHTS PRELIMS TEST SERIES 2017

www.insightsonindia.com www.insightsias.com
TEST 4 Solutions

Statement 2: These forests are not owned by private citizens or


companies. State governments are authorized to declare such areas as
reserved or protected forests depending on the conservation requirement
of the region.

Q Source: Improvisation: Page 20: Xth Geography NCERT:


Contemporary India - II

44. Consider the following matches of movements with the


region they were started in.
1. Chipko movement : Himalayas
2. Beej Bachao Andolan : Madhya Pradesh
3. Navdanya movement : Ladakh

Select the correct answer using the codes below.

a) 1 and 2 only
b) 1 only
c) 2 and 3 only
d) 1, 2 and 3

Solution: b)
Learning: The famous Chipko movement in the Uttarakhand
Himalayas has not only successfully resisted deforestation in several
areas but has also shown that community afforestation with indigenous
species can be enormously successful.

Farmers and citizens groups like the Beej Bachao Andolan in Tehri
(Uttarakhand) and Navdanya (scattered over several states) have
shown that adequate levels of diversified crop production without
the use of synthetic chemicals are possible and economically
viable.
Navdanya is a women centred movement for the protection of
biological and cultural diversity.


INSIGHTS PRELIMS TEST SERIES 2017

www.insightsonindia.com www.insightsias.com
TEST 4 Solutions

Navdanya is a network of seed keepers and organic producers


spread across 18 states in India.

Q Source: Page 21: Xth Geography NCERT: Contemporary India - II

45. Gross Domestic Product of India is


a) Total capital investment made in the country within an year
b) Total value of goods and services produced by its nationals
c) Total consumption expenditure of all residents of India
d) None of the above

Solution: d)
Justification: It is the value of the final goods and services produced
within a country within an year.

Option C is wrong, since consumption also include imports from abroad,


and all residents are not citizens. Moreover, consumption expenditure is
only one component of GDP.

Option B is wrong, since Indian nationals living abroad do not contribute


to Indias GDP. They do however contribute to Indias GNP.

Option A is wrong as investment is just one component of GDP.

Q Source: Improvisation: Page 23: Standard Xth Economics NCERT:


Understanding Economic Development

46. If you happen to take the shortest aerial route between


Uttarakhand and Odisha, which of the following major dams will
lie eastwards from your route?
1. Panchet


INSIGHTS PRELIMS TEST SERIES 2017

www.insightsonindia.com www.insightsias.com
TEST 4 Solutions

2. Mettur
3. Rana-pratap Sagar

Select the correct answer using the codes below.

a) 1 and 2 only
b) 1 only
c) 2 and 3 only
d) 3 only

Solution: b)
Justification: Statement 1: It was constructed across the Damodar
River at Panchet in Dhanbad district in Jharkhand. It will lie eastwards
en route. So, 1 is correct.

Statement 2: Mettur dam is in South India, and clearly cant be the


answer. So, 2 is incorrect.

Statement 3: Ranapratap Sagar Dam is built on the Chambal River at


Rawatbhata in Rajasthan. It cant lie en route.

See the map for reference.


INSIGHTS PRELIMS TEST SERIES 2017

www.insightsonindia.com www.insightsias.com
TEST 4 Solutions

Q Source: Improvisation: Page 28: Xth Geography NCERT:


Contemporary India - II

47. The major iron ore belts in India are


1. Orissa-Jharkhand belt
2. Durg-Bastar-Chandrapur belt


INSIGHTS PRELIMS TEST SERIES 2017

www.insightsonindia.com www.insightsias.com
TEST 4 Solutions

3. Bellary-Chitradurga-Chikmaglur-Tumkur belt

Select the correct answer using the codes below.

a) 1 and 2 only
b) 1 only
c) 2 and 3 only
d) 1, 2 and 3

Solution: d)
Justification: Statement 1: In Orissa high grade hematite ore is found
in Badampahar mines in the Mayurbhanj and Kendujhar districts. In the
adjoining Singbhum district of Jharkhand haematite iron ore is mined in
Gua and Noamundi.

Statement 2: It lies in Chhattisgarh and Maharashtra. Very high grade


hematites are found in the famous Bailadila range of hills in the Bastar
district of Chattisgarh.

Statement 3: It is located in Karnataka.

Learning: Maharashtra-Goa belt includes the state of Goa and


Ratnagiri district of Maharashtra. Though, the ores are not of ery high
quality, yet they are efficiently exploited. Iron ore is exported through
Marmagao port.

The Kudermukh mines located in the Western Ghats of Karnataka are a


100 per cent export unit. Kudremukh deposits are known to be one of
the largest in the world

Q Source: Page 53: Xth Geography NCERT: Contemporary India - II


INSIGHTS PRELIMS TEST SERIES 2017

www.insightsonindia.com www.insightsias.com
TEST 4 Solutions

48. India has three cropping seasons rabi, kharif and zaid.
Consider the following about them.
1. Zaid crops are sown at the start of winter and harvested in
summer.
2. Kharif crops are grown with the onset of monsoon.
3. Gram and Mustard are important Rabi crops.
4. Rabi crops cannot be grown without irrigation facilities.

Select the correct answer using the codes below.

a) 1, 2 and 4 only
b) 1, 2 and 3 only
c) 2 and 3 only
d) 1 and 4 only

Solution: c)
Justification & Learning: Statement 1: Zaid Season lies between
Rabi and Kharif Seasons. It is the Rabi season that starts in Winter
around October.

Statement 2: These are sown when Monsoon nears and harvested in


September-October. Important crops grown during this season are
paddy, maize, jowar, bajra, tur (arhar), moong, urad, cotton, jute,
groundnut and soyabean.

Statement 3: Some of the important rabi crops are wheat, barley, peas,
gram and mustard.

Though, these crops are grown in large parts of India, states from the
north and northwestern parts such as Punjab, Haryana, Himachal
Pradesh, Jammu and Kashmir, Uttaranchal and Uttar Pradesh are
important for the production of wheat and other rabi crops.

Statement 4: Irrigation is not absolutely vital for these crops. It however


helps improve the productivity.

Availability of precipitation during winter months due to the western


temperate cyclones helps in the success of these crops.

Q Source: Page 36: Xth Geography NCERT: Contemporary India - II


INSIGHTS PRELIMS TEST SERIES 2017

www.insightsonindia.com www.insightsias.com
TEST 4 Solutions

49. Kaiser-i-Hind in its natural habitat can be found in


a) Eastern Himalayas
b) Western Salt marshes
c) Southern Western Ghats
d) Deciduous forests of Eastern India

Solution: a)
Learning: It is called The Emperor of India.

This magnificent butterfly is found along the Eastern Himalayas (West


Bengal, Meghalaya, Assam, Sikkim and Manipur) in India.

It has a strong and fast flight. It usually flies at tree-top level but when
there is strong morning sunlight it descends to sit on low vegetation.

Although the butterfly is protected under Indian law it is hunted to be


supplied to butterfly collectors.


INSIGHTS PRELIMS TEST SERIES 2017

www.insightsonindia.com www.insightsias.com
TEST 4 Solutions

Q Source:
http://www.wwfindia.org/about_wwf/priority_species/lesser_known_s
pecies/kaiser_i_hind/

50. Which of the following conditions facilitate the growth of


Jute?
a) Well-drained fertile soils in the flood plains and high Temperature
at the time of growth
b) Water retaining soils in hilly areas where mean annual
temperature is generally lower than plains
c) Sandy soils in arid and semi-arid regions
d) Laterite soils in Western Coastal regions of India within moderate
temperature zones

Solution: a)
Learning: West Bengal, Bihar, Assam, Orissa and Meghalaya are the
major jute producing states.

Jute grows well on well-drained fertile soils in the flood plains where
soils are renewed every year. High temperature is required during the
time of growth.

It is known as the golden fibre. It is used in making gunny bags, mats,


ropes, yarn, carpets and other artefacts. Due to its high cost, it is losing
market to synthetic fibres and packing materials, particularly the nylon.

Q Source: Page 42: Xth Geography NCERT: Contemporary India - II


INSIGHTS PRELIMS TEST SERIES 2017

www.insightsonindia.com www.insightsias.com
TEST 4 Solutions

51. Gross Enrolment Ratio (GER) is a statistical tool widely used to


show the general level of participation in
a) Primary Education only
b) Secondary Education only
c) Primary and Secondary Education only
d) Primary, Secondary and Tertiary Education

Solution: d)
Learning: The data is defined as the percentage of student enrolment
in primary and secondary schools (and in tertiary educational
institutions) on gender basis.

Gross Enrolment Ratio (GER) in Higher education in India is calculated


for 18-23 years of age group.

Gender parity index is the ratio of Girls GER to Boys GER in a given
population.

Outbound Mobility Ratio is the total number of students from a given


country studying abroad, expressed as a percentage of total tertiary
enrolment in that country.

http://mhrd.gov.in/sites/upload_files/mhrd/files/statistics/EAG2014.p
df

Q Source: Improvisation: Page 2: Standard Xth Economics NCERT:


Understanding Economic Development

52. Consider the following about Indian Council of Agricultural


Research (ICAR).
1. It was established before independence in pursuance of the
report of the Royal Commission on Agriculture.
2. It is an autonomous organisation under the Department of
Farmers Welfare (DFW).

Which of the above is/are correct?


INSIGHTS PRELIMS TEST SERIES 2017

www.insightsonindia.com www.insightsias.com
TEST 4 Solutions

a) 1 only
b) 2 only
c) Both 1 and 2
d) None

Solution: a)
Justification: Statement 1: It is an autonomous organisation under the
Department of Agricultural Research and Education (DARE), Ministry of
Agriculture and Farmers Welfare, Government of India.

Statement 2: Formerly known as Imperial Council of Agricultural


Research, it was established in 1929 as a registered society under the
Societies Registration Act, 1860 in pursuance of the report of the Royal
Commission on Agriculture.

The ICAR has its headquarters at New Delhi.

Learning: The Council is the apex body for co-ordinating, guiding and
managing research and education in agriculture including horticulture,
fisheries and animal sciences in the entire country.

The ICAR has played a pioneering role in ushering Green Revolution and
subsequent developments in agriculture in India.

Q Source: Improvisation: Page 44: Xth Geography NCERT:


Contemporary India - II

53. Human Development Report published by United Nations


Development Programme (UNDP) compares countries based on
their
1. Income levels
2. Educational attainments
3. Health status

Select the correct answer using the codes below.


INSIGHTS PRELIMS TEST SERIES 2017

www.insightsonindia.com www.insightsias.com
TEST 4 Solutions

a) 1 and 2 only
b) 3 only
c) 1 only
d) 1, 2 and 3

Solution: d)
Justification: Over the past few decades, health and education
indicators have come to be widely used along with income as a measure
of development.

For instance, Human Development Report published by UNDP


compares countries based on the educational levels of the people, their
health status and per capita income.

As per the UNDP reports, India is lagging behind in HDI from its
neighbour Sri Lanka and some other close by like Saudi Arabia.

This tells us that economic growth is not the only important determinant
of human development, and encompasses various other indicators like
social status of different communities, distribution of wealth and
empowerment.

Q Source: Page 13: Standard Xth Economics NCERT: Understanding


Economic Development

54. Genetic Engineering can be used to modify


1. Plant cells
2. Animals cells
3. Microorganisms including Virus

Select the correct answer using the codes below.

a) 1 and 2 only
b) 1 only
c) 2 and 3 only
d) 1, 2 and 3


INSIGHTS PRELIMS TEST SERIES 2017

www.insightsonindia.com www.insightsias.com
TEST 4 Solutions

Solution: d)
Justification: Statement 1: Plants have been modified for insect
protection, herbicide resistance, virus resistance, enhanced nutrition,
tolerance to environmental pressures and the production of edible
vaccines, for e.g. Bt cotton, Bt Mustard etc.

Statement 2: Genetic engineering is used to create animal models of


human diseases. Genetically modified mice are the most common
genetically engineered animal model.

Statement 3: Bacteria were the first organisms to be genetically


modified.

In medicine, genetic engineering has been used in manufacturing drugs,


to create model animals and do laboratory research, and in gene therapy.
Genetically engineered viruses are being developed that can still confer
immunity, but lack infectious sequences.

Q Source: Improvisation: Page 47: Xth Geography NCERT:


Contemporary India - II

55. Consider the following about National Payments Corporation


of India (NPCI).
1. It is a not-for-profit organisation.
2. It is the umbrella organisation for all retail payments system in
India.
3. It conceived and launched the domestic card payment network
RuPay.

Select the correct answer using the codes below.

a) 1 and 2 only
b) 2 and 3 only
c) 1 only


INSIGHTS PRELIMS TEST SERIES 2017

www.insightsonindia.com www.insightsias.com
TEST 4 Solutions

d) 1, 2 and 3

Solution: d)
Justification: NPCI is the umbrella organisation for all retail payments
system in India.

It is being promoted by the Reserve Bank of India and founded in 2008


as a not-for-profit organisation registered under section 25 of the
Companies Act, 2013.

It has successfully played pioneering role in the development of a


domestic card payment network called RuPay, reducing the dependency
on international card schemes.

Learning: RuPay facilitates electronic payment at all Indian banks and


financial institutions, and competes with MasterCard and Visa in India.

Banks in India are authorized to issue RuPay debit cards to their


customers for use at ATMs, PoS terminals, and e-commerce websites.

RuPay cards got a major boost through the Pradhan Mantri Jan Dhan
Yojana as RuPay cards were issued to all Jan Dhan accounts.

Q Source: Revision Past Test Syllabus: Current Affairs

56. Ground water remains underexploited in which of the


following regions in India?
a) Agriculturally prosperous regions of Punjab and Western U.P
b) Hard rock plateau areas of Central and Southern India
c) All coastal areas
d) There is evidence of heavy exploitation, not under exploitation, in
some of the above mentioned areas.

Solution: d)
Learning: Recent evidence suggests that the groundwater is under
serious threat of overuse in many parts of the country.


INSIGHTS PRELIMS TEST SERIES 2017

www.insightsonindia.com www.insightsias.com
TEST 4 Solutions

About 300 districts have reported a water level decline of over 4


metres during the past 20 years. Nearly one-third of the country is
overusing their groundwater reserves.

In another 25 years, 60 per cent of the country would be doing the


same if the present way of using this resource continues.

Groundwater overuse is particularly found in the agriculturally


prosperous regions of Punjab and Western U.P., hard rock plateau
areas of central and south India, some coastal areas and the rapidly
growing urban settlements.

Ground water is overused due to free electricity, non-regulation of


illegal borewells, growing highly water-intensive crops and
pollution of surface water reserves in many areas.

Q Source: Improvisation: Page 14: Standard Xth Economics NCERT:


Understanding Economic Development

57. Consider the following statements.


1. Balaghat mines in Madhya Pradesh are known for copper
production.
2. Khetri mines in Rajasthan are famous as a major aluminium
producer.

Which of the above is/are correct?

a) 1 only
b) 2 only
c) Both 1 and 2
d) None

Solution: a)


INSIGHTS PRELIMS TEST SERIES 2017

www.insightsonindia.com www.insightsias.com
TEST 4 Solutions

Justification: The Balaghat mines in Madhya Pradesh produce 52 per


cent of Indias copper.

The Singbhum district of Jharkhand is also a leading producer of copper.

The Khetri mines in Rajasthan are also famous for Copper, not
Aluminium. In fact, rajasthan is not even a major producer of
Aluminium. So, 2 is wrong.

Q Source: Page : Xth Geography NCERT: Contemporary India - II

58. Consider the following about CUTS International (Consumer


Unity & Trust Society).
1. It is the Indian subsidiary of Consumers International (CI), a
non-profit organization.
2. It works as an autonomous attached advisory agency of the
Ministry of Consumer Affairs and Public Distribution.

Which of the above is/are correct?

a) 1 only
b) 2 only
c) Both 1 and 2
d) None

Solution: d)
Justification: Statement 1: It is an Indian NGO (now spread in several
nations) started from Rajasthan.

Consumers International is an independent international NGO.

Statement 2: The organisations Vision is Consumer Sovereignty. It


conducts action (policy) research and evidence-based advocacy for policy
and practice changes to bridge the gaps between the core and the
periphery, that between the state and non-state actors.


INSIGHTS PRELIMS TEST SERIES 2017

www.insightsonindia.com www.insightsias.com
TEST 4 Solutions

Q Source: Improvisation: Page 74 & 78: Standard Xth Economics


NCERT: Understanding Economic Development

59. Petroleum occurrences in India are generally associated with


1. Anticlines and fault traps in the rock formations of the tertiary
age.
2. Porous limestone or sandstone layers within Crust

Which of the above is/are correct?

a) 1 only
b) 2 only
c) Both 1 and 2
d) None

Solution: c)
Justification: Syncline and anticline are terms used to describe folds
based on the relative ages of folded rock layers. A syncline is a fold in
which the youngest rocks occur in the core of a fold (i.e., closest to the
fold axis), whereas the oldest rocks occur in the core of an anticline.

In regions of folding, anticlines or domes, it occurs where oil is trapped


in the crest of the upfold. The oil bearing layer is a porous limestone or
sandstone through which oil may flow.

Learning: The oil is prevented from rising or sinking by intervening


non-porous layers.

Petroleum is also found in fault traps between porous and non-porous


rocks. Gas, being lighter usually occurs above the oil.

About 63 per cent of Indias petroleum production is from Mumbai High,


18 per cent from Gujarat and 16 per cent from Assam.

Q Source: Page 60: Xth Geography NCERT: Contemporary India - II


INSIGHTS PRELIMS TEST SERIES 2017

www.insightsonindia.com www.insightsias.com
TEST 4 Solutions

60. The Union Finance Ministry has recently issued fresh flexi-
fund guidelines for the Centrally Sponsored Schemes (CSS). Flexi-
funds are those funds which can be used by states to
1. Bridge the revenue expenditure gap in the State budget
2. Meet local development requirements that are in line with the
overall goal of the CSS

Which of the above is/are correct?

a) 1 only
b) 2 only
c) Both 1 and 2
d) None

Solution: b)
Justification: Statement 1: Flexi funds are not the normal statutory or
discretionary grants given by the Centre. They are a component of the
CSS support received by the States. With the exception of CSS, the
funding under CSS is to be strictly spent on the implementation of that
particular CSS only.

Statement 2: Under the new norms, flexi-funds in each CSS has been
increased from the current 10% to 25% for states and 30% for Union
Territories.

This means that states if they so desire can set aside 25% of any
CSS as flexi-fund to be spent on any sub-scheme or innovation or
component that is in line with the overall aim and objective of the
approved Centrally Sponsored Scheme.

States can use the fund to satisfy local requirements in areas


affected by internal security disturbances or to undertake
mitigation or restoration activities in case of natural calamities.

State governments will have to constitute a state-level sanctioning


committee (SLSC) to avail of the flexi-fund facility.


INSIGHTS PRELIMS TEST SERIES 2017

www.insightsonindia.com www.insightsias.com
TEST 4 Solutions

Q Source:
http://finmin.nic.in/the_ministry/dept_expenditure/notification/gaasp
/Guidelines_FlexiFunds_Schemes.pdf

61.The Village Roads (VRs) and Other District Roads (ODRs), which
are often referred as Rural Roads are maintained by
a) Public Works Departments or Rural Development Departments
within the State government
b) State Agricultural marketing boards
c) Panchayati Raj Institutions (PRIs)
d) Any of the above depending on the rural road network

Solution: d)
Learning: A wide range of government institutions are building rural
roads in India.

The Village Roads (VRs) and Other District Roads (ODRs), two
categories of roads referred as Rural Roads, are normally under the
jurisdiction of the Public Works Departments or Rural Development
Departments within the State government administrations.

There are also various agricultural marketing boards and similar


agencies mandated to build and maintain rural roads.

In addition, the Panchayati Raj Institutions at local government levels


are in charge of some portions of the rural road network.

Q Source: Improvisation: Page 82: Xth Geography NCERT:


Contemporary India - II

62. Tyndall effect can be observed when


1. Sunlight passes through the canopy of a dense forest
2. A fine beam of light enters a room through a small hole


INSIGHTS PRELIMS TEST SERIES 2017

www.insightsonindia.com www.insightsias.com
TEST 4 Solutions

Which of the above is/are correct?

a) 1 only
b) 2 only
c) Both 1 and 2
d) None

Solution: c)
Justification: The small sized colloidal particles in air can easily scatter
a beam of visible light. This scattering of a beam of light is called the
Tyndall effect.

Statement 1: In the forest, mist contains tiny droplets of water, which


act as particles of colloid dispersed in air.

Statement 2: Dust and smoke present in the air scatter the beam of light.

Q Source: Page 18: IXth Science NCERT

63. Consider the following statements about the views of Ancient


Indian philosophers.
1. Maharishi Kanad gave the idea of Atom as an indivisible part of
matter.
2. Pakudha Katyayama gave the idea that the Universe is an
indivisible whole and atoms do not have any real existence.

Which of the above is/are correct?

a) 1 only
b) 2 only
c) Both 1 and 2
d) None

Solution: a)
Justification: Statement 1: The idea of divisibility of matter was
considered long back in India, around 500 BC. An Indian philosopher


INSIGHTS PRELIMS TEST SERIES 2017

www.insightsonindia.com www.insightsias.com
TEST 4 Solutions

Maharishi Kanad, postulated that if we go on dividing matter (padarth),


we shall get smaller and smaller particles.

Ultimately, a time will come when we shall come across the smallest
particles beyond which further division will not be possible.

He named these particles Parmanu.

Statement 2: Another Indian philosopher, Pakudha Katyayama,


elaborated this doctrine and said that these particles normally exist in a
combined form which gives us various forms of matter.

Q Source: Page 31: IXth Science NCERT

64. If India finds that China is oversubsidizing its solar cell


manufacturing industry to boost its exports, which of following
actions can be taken by India?
a) India can move the WTO citing violation of trade rules.
b) India can raise the dispute under the United Nations General
Assembly (UNGA).
c) India can drag China to the United Nations Economic and
Social Council (UNECOSOC) to settle the dispute.
d) India cannot take any action against China as the latter is not
a member of WTO.

Solution: a)
Justification: UNECOSOC and UNGA do not settle trade related
dispute between its member parties. So, B and C are wrong.

But, the complaint can be made to WTO as China is its member. So, D is
wrong.

For e.g. India has recently complained to the World Trade Organization
(WTO) about support given to the renewable energy industry in United
States.


INSIGHTS PRELIMS TEST SERIES 2017

www.insightsonindia.com www.insightsias.com
TEST 4 Solutions

Learning: The complaint alleges some states in the USA have prop up
their renewable sector with illegal subsidies and domestic content
requirements, an obligation to buy local goods rather than imports.

By filing the complaint, India has triggered a 60-day window for US to


settle the dispute, after which India could ask the WTO to adjudicate.

Q Source: Improvisation: Current Affairs: India-USA WTO dispute

65. The Union Cabinet has approved establishment of Higher


Education Financing Agency (HEFA) to
1. Provide credit support to meritorious students who cannot
afford tuition fees of premier Central Institutions
2. Create high quality infrastructure in premier educational
institutions

Which of the above is/are correct?

a) 1 only
b) 2 only
c) Both 1 and 2
d) None

Solution: b)
Justification: The HEFA will be jointly promoted by the Union
Ministry of Human Resource Development (MHRD) and identified
Promoter.

The HEFA will also mobilise CSR funds from Corporates/PSUs


which will in turn be released for promoting research and
innovation in these institutions on grant basis.

All the Centrally Funded Higher Educational Institutions will be


eligible to join as members of the HEFA.


INSIGHTS PRELIMS TEST SERIES 2017

www.insightsonindia.com www.insightsias.com
TEST 4 Solutions

It would leverage the equity to raise up to Rs. 20,000 crore for


funding infrastructure and development projects of world class
Labs in IIMs/IITs/NITs and such other institutions.

The principal portion of the loan will be repaid through the


internal accruals of the institutions earned through the fee
receipts, research earnings etc.

Q Source:
http://timesofindia.indiatimes.com/home/education/Cabinet-gives-
nod-to-Higher-Education-Financing-Agency-
HEFA/articleshow/54291736.cms

66. The classification of Organised and unorganized sector in


India is mainly based on
a) Employment conditions
b) Working duration of the firm
c) Gross output of the firm
d) Net profits of the firm after paying taxes

Solution: a)
Learning: Sectors are majorly divided into three categories primary,
secondary and tertiary. Based on the employment conditions these are
further classified as organised and unorganised sector.

The sector, which is registered with the government is called an


organised sector. The sector which is not registered with the government
and whose terms of employment are not fixed and regular is considered
as unorganised sector.


INSIGHTS PRELIMS TEST SERIES 2017

www.insightsonindia.com www.insightsias.com
TEST 4 Solutions

Q Source: Page 31: Standard Xth Economics NCERT: Understanding


Economic Development

67. Which of the following neighbouring countries performs


better in terms of Human Development Index (HDI) than India?
1. Bangladesh


INSIGHTS PRELIMS TEST SERIES 2017

www.insightsonindia.com www.insightsias.com
TEST 4 Solutions

2. Sri Lanka
3. Nepal
4. Pakistan

Select the correct answer using the codes below.

a) 1 and 2 only
b) 1 and 4 only
c) 2 only
d) 1, 2 and 3 only

Solution: c)
Justification: HDI of these nations are:

Sri Lanka 0.757

India 0.609

Pakistan 0.538

Nepal 0.548

Bangladesh0.570

Bangladesh, Nepal and Pakistan are classified under Low Human


Development category. India and SL are under the Medium Human
Development category.

India is placed at 130th rank and Pakistan, 147th. As for the growth rate
during 1990-2014, South Asias figure was 1.38, the highest among all
regions.

Q Source: Chapter 1: Standard Xth Economics NCERT: Understanding


Economic Development

68. Forest Spotted Owlet is


INSIGHTS PRELIMS TEST SERIES 2017

www.insightsonindia.com www.insightsias.com
TEST 4 Solutions

1. A Critically Endangered species


2. Endemic to North-Western Himalayas

Which of the above is/are correct?

a) 1 only
b) 2 only
c) Both 1 and 2
d) None

Solution: a)
Justification: The forest owlet (Athene blewitti) is an owl that is
endemic to the forests of central India.

The forest owlet has sightings from the Talda Forest Range, the
Toranmal Forest Range, the Melghat Tiger Reserve, and the Khaknaar
Forest Range, all in central India had dense to open deciduous forest.

Forest in its range is being lost and degraded by illegal tree cutting for
firewood and timber, and encroachment for cultivation, grazing and
settlements as well as forest fires and minor irrigation dams

Q Source: Page 15: Xth Geography NCERT: Contemporary India - II

69. What is Internet of Things (IoT)?


a) It is the internetworking of physical devices and objects to
communicate without requiring human interference.
b) It is a more secure form of Virtual Private Network (VPN).
c) It is a centralized surveillance system for monitoring internet
traffic.
d) It is a common domain that will host all mother sites on the
web.

Solution: a)


INSIGHTS PRELIMS TEST SERIES 2017

www.insightsonindia.com www.insightsias.com
TEST 4 Solutions

Learning: IoT is a system of interrelated computing devices,


mechanical and digital machines, objects, or people that are provided
with unique identifiers.

Thus it can be said that it is an internetworking of physical devices,


vehicles, buildings and other itemsembedded with electronics,
software, sensors and network connectivity.

The internetworking has ability to transfer data over a network without


requiring human-to-human or human-to-computer interaction.

IoT is also dubbed as the infrastructure of the information society. It


allows objects to be sensed and controlled remotely across existing
network infrastructure.

You can see some applications here


http://www.postscapes.com/internet-of-things-examples/

Q Source: The first edition of IoT (Internet of Things) India Congress,


2016

70. India has signed a Open Skies Memorandum of


Understanding (MoU) with Greece. How will the MoU be helpful?
a) It will allow reduce the limit on the number of flights into
each others territory.
b) All airline passengers from both countries will be given
hassle free Visa on Arrival (VoA).
c) Trade through air route shall be exempt from paying duties.
d) All of the above

Solution: a)
Justification: At present, there are no direct flights between India and
Greece. Passengers have to travel one stop via Turkey or Gulf States to
Greece.


INSIGHTS PRELIMS TEST SERIES 2017

www.insightsonindia.com www.insightsias.com
TEST 4 Solutions

Till now there was no air services agreement between India and
Greece and thus preventing the start of air services between two
countries.

Under the open skies agreement, airlines from India will be


allowed to operate unlimited number of flights to Greece.

Similarly airlines carriers from Greek will be granted unlimited


traffic rights to six Indian metro cities.

Learning: Under the new civil aviation policy, India plans to enter into
open sky air service agreements (ASA) with SAARC countries and with
countries beyond 5,000 km radius from Delhi.

At present, India has an open sky agreement with the US and a near
open sky agreement with the UK.

Q Source:
http://economictimes.indiatimes.com/industry/transportation/airlines-
/-aviation/india-signs-open-skies-agreement-with-
greece/articleshow/54048405.cms

71. Consider the following about Paralympics.


1. They were organized for the first time in 2012 Olympics.
2. The International Paralympic Committee (IPC) is the global
governing body of the Paralympic Movement.

Which of the above is/are correct?

a) 1 only
b) 2 only
c) Both 1 and 2
d) None

Solution: b)


INSIGHTS PRELIMS TEST SERIES 2017

www.insightsonindia.com www.insightsias.com
TEST 4 Solutions

Justification: Statement 1: They were first held in Rome, Italy in 1960


and featured 400 athletes from 23 countries competing in 13 sports.

Since then the Games have taken place every four years in the same year
as the Olympic Games.

Statement 2: Its purpose is to organise the summer and winter


Paralympic Games and act as the International Federation for nine
sports, supervising and coordinating World Championships and other
competitions.

It was founded in 1989 as a non-profit organisation. It is based in Bonn,


Germany and aims to develop sports opportunities for all people with
impairment from the beginner to elite level.

Q Source: https://www.paralympic.org/the-ipc/paralympic-games

72. The rupee is accepted as a medium of exchange mainly


because
1. Indian law legalises the use of rupee as a medium of payment.
2. Money supply in form of rupee is unlimited.
3. Production cost of a rupee is equal to its value.

Select the correct answer using the codes below.

a) 1 and 2 only
b) 2 and 3 only
c) 1 only
d) 1 and 3 only

Solution: c)
Justification: Statement 1: Rupee is authorised as a medium of
exchange by the RBI on behalf of the government based on law. As per
Indian law, no other individual or organisation is allowed to issue
currency. Hence, it is widely accepted as a medium of exchange.

Statement 2: While rupee currency notes can theoretically be printed on


a large scale, but this is not the reason why it is accepted as a medium of


INSIGHTS PRELIMS TEST SERIES 2017

www.insightsonindia.com www.insightsias.com
TEST 4 Solutions

exchange. Many commodities in India can be supplied on a large scale,


for e.g. water. Wide supply of a commodity does not necessarily lead to
its establishment as the medium of exchange of the country.

Statement 3: For e.g. a 1000 Rupees intrinsic value (production cost) is


not even a fraction of its actually value in market. So, 3 is incorrect.

Q Source: Page 40: Standard Xth Economics NCERT: Understanding


Economic Development

73. Which of the following countries are member(s) of G-20?


1. Australia
2. Saudi Arabia
3. Canada
4. China
5. Japan

Select the correct answer using the codes below.

a) 1, 3 and 5 only
b) 1, 2 and 4 only
c) 2, 3 and 5 only
d) 1, 2, 3, 4 and 5

Solution: d)
Justification: Use elimination. China is an influential member of G-20.
So, options A and C can be discarded right away.

There is no reason why Japan, being a major developed economy, should


not be a member of G-20. This clearly leads to the answer option D.

Learning: Members are:


INSIGHTS PRELIMS TEST SERIES 2017

www.insightsonindia.com www.insightsias.com
TEST 4 Solutions

G20 is an international forum for the governments and central


bank governors from 20 major economies.

It was founded in 1999 with the aim of studying, reviewing, and


promoting high-level discussion of policy issues pertaining to the
promotion of international financial stability.

It seeks to address issues that go beyond the responsibilities of any


one organization.

The G20 heads of government or heads of state have periodically


conferred at summits since their initial meeting in 2008, and the
group also hosts separate meetings of finance ministers and central
bank governors.

Q Source: G-20 summit China 2016

74. How is China-Pakistan economic corridor (CPEC) important


to China?
1. The project will shorten the route for Chinas energy imports
from the Middle East.
2. It will strengthen Chinese presence in the Arabian Ocean.

Which of the above is/are correct?

a) 1 only
b) 2 only
c) Both 1 and 2
d) None


INSIGHTS PRELIMS TEST SERIES 2017

www.insightsonindia.com www.insightsias.com
TEST 4 Solutions

Solution: c)
Justification: Statement 1: The Straits of Malacca provide China with
its shortest maritime access to Europe, Africa, and the Middle East.

Approximately 80% pass of its Middle Eastern energy imports also


pass through the Straits of Malacca.

In addition to vulnerabilities faced in the Straits of Malacca region,


China is heavily dependent upon sea-routes that pass through the
South China Sea, near the disputed Spratly Islands and Paracel
Islands.

The CPEC project will allow Chinese energy imports to circumvent


these contentious areas and find a new artery in the west, and
thereby decrease the possibility of confrontation between the
United States and China.

Statement 2: CPEC will connect Xinjiang (in China) with Pakistans


Gwadar Port (located at the shores of Arabian Ocean) through PoK.

Growing Chinese presence in western neighbourhood is a strategic


concern for India.

Q Source: http://timesofindia.indiatimes.com/world/china/Chinese-
daily-warns-of-setbacks-to-China-Pakistan-economic-
corridor/articleshow/54319019.cms?

75. Consider the following about National Infrastructure and


Investment Fund (NIIF).
1. There will no contribution by the government in the fund.
2. The Fund aims to attract investment from both domestic and
international sources.
3. NIIF will also fund Centrally Sponsored Schemes (CSS) in
Indian states as a part of its Corporate Social Responsibility
(CSR).


INSIGHTS PRELIMS TEST SERIES 2017

www.insightsonindia.com www.insightsias.com
TEST 4 Solutions

Select the correct answer using the codes below.

a) 1 and 2 only
b) 2 only
c) 1 and 3 only
d) 3 only

Solution: b)
Justification: The Union Cabinet, in July 2015, had approved creation
of National Infrastructure and Investment Fund (NIIF) as a trust that
will have a corpus of Rs 20,000 crore.

Statement 1: The governments contribution would be limited to 49% of


the subscribed capital.

Statement 2: The government will seek participation from strategic


investors such as sovereign fund, quasi sovereign funds and multilateral
or bilateral investors, which can help leverage this fund to many times.

Cash-rich PSUs, pension funds, provident funds, National Small


Saving Fund will be able to pick up stake in the fund.

Statement 3: The objective of NIIF is to maximize economic impact


mainly through infrastructure development in commercially viable
projects, both greenfield and brownfield, including stalled projects.

It will not be investing in social welfare schemes.

Learning: The functions of NIIF are as follows:

Fund raising through suitable instruments including off-shore


credit enhanced bonds, and attracting anchor investors to
participate as partners in NIIF;

Servicing of the investors of NIIF.

Considering and approving candidate companies/institutions/


projects (including state entities) for investments and periodic
monitoring of investments.

Investing in the corpus created by Asset Management Companies


(AMCs) for investing in private equity.


INSIGHTS PRELIMS TEST SERIES 2017

www.insightsonindia.com www.insightsias.com
TEST 4 Solutions

Preparing a shelf of infrastructure projects and providing advisory


services.

Q Source: http://www.insightsonindia.com/2016/09/14/insights-
daily-current-affairs-14-september-2016/

76. The severe decline in the birds population is largely


attributed to the degradation of riverine habitats including loss of
riparian forest corridors. The bird is found in Assam and
Arunachal Pradesh. It is?
a) White Winged Wood Duck
b) Slender Pigeon
c) Barb
d) Green headed Pigeon

Solution: a)
Learning: It is one of the most endangered birds. In India, the duck is
limited to Assam and Arunachal Pradesh. Due to its ghostly call, it is
called Deo Hans or Spirit Duck in Assamese.

It mostly resides in dense tropical evergreen forest and is known to


prefer inaccessible swampy areas formed by numerous rivers, streams,
etc.

WWF-India is working to conserve the habitats of white winged wood


duck though its Western Arunachal Pradesh and North Bank Landscapes
programmes in Arunachal Pradesh and Assam respectively.

Q Source: Test Syllabus:


http://www.wwfindia.org/about_wwf/priority_species/lesser_known_s
pecies/wood_duck/


INSIGHTS PRELIMS TEST SERIES 2017

www.insightsonindia.com www.insightsias.com
TEST 4 Solutions

77.Heavy industries and thermal power station are located on or near


the coalfields because
1. Coal is a weight losing raw material.
2. Tertiary coal deposits in India occur only near the river beds.

Which of the above is/are correct?

a) 1 only
b) 2 only
c) Both 1 and 2
d) None

Solution: a)
Justification: Statement 1: Coal is a bulky material, which loses weight
on use as it is reduced to ash. So, it is not profitable for companies to
transport coal to long distances as a lot of coal content is not useful.
Locating near the coalfields saves on such costs. So, 1 is correct.

Statement 2: Coal in India is found in Gondawana deposits in the


Eastern belt, and Tertiary belts in North-east. Tertiary belt is not
restricted to river beds.

Q Source: Page 59-60: Xth Geography NCERT: Contemporary India -


II

78. Which of the following about Ganges Shark is INCORRECT?


a) It is endemic to India.
b) Large populations of Ganges Shark can be found in east
flowing distributaries of Ganga as well Eastern Indian Ocean.
c) It is protected under Schedule I of the Wildlife (Protection)
Act, 1972 and listed as a Critically Endangered species in the
IUCN Redlist.


INSIGHTS PRELIMS TEST SERIES 2017

www.insightsonindia.com www.insightsias.com
TEST 4 Solutions

d) None of the above

Solution: b)
Justification & Learning: There are six species of river sharks found
in the world, out of which the Ganges shark (Glyphis gangeticus) is
endemic to India. So, A is correct.

It inhabits the River Hooghly in West Bengal, as well as the rivers


Ganges, Brahmaputra, Mahanadi in the states of Bihar, Assam and
Orissa.

While some of the other river sharks are also known to inhabit saltwater,
the Ganges shark is only found in rivers and possibly estuaries, with no
confirmed records from oceans or seas. So, option B is incorrect.

Learning: Its population has been steadily decreasing due to over


fishing, habitat degradation, increasing river utilisation, and building of
dams. Its fin and jaws are in high demand in the international trade, and
is also fished by locals for its meat and oil.

Q Source:
http://www.wwfindia.org/about_wwf/priority_species/lesser_known_s
pecies/ganges_shark/

79. Consider the following statements about various


organizations dealing with statistics in India.
1. NSSO is an organisation under the Ministry of Statistics,
Planning and Programme Implementation.
2. National Statistical Commission (NSC) was setup on the
recommendations of the Rangarajan Commission.
3. Central Statistical Office (CSO) conducts Annual Survey of
Industries.

Select the correct answer using the codes below.

a) 1 and 2 only


INSIGHTS PRELIMS TEST SERIES 2017

www.insightsonindia.com www.insightsias.com
TEST 4 Solutions

b) 2 only
c) 3 only
d) 1, 2 and 3

Solution: d)
Justification: Statement 1: It is the largest organisation in India
conducting regular socio-economic surveys. It was established in 1950.

Statement 2: The Commission (setup in 2005) has a part-time


Chairperson, four part-time Members and an ex-officio Member. The
Chief Statistician of India, the post created specifically as the Head of the
National Statistical Office is the Secretary of the Commission

Statement 3: Activities of CSO include compilation of National


Accounts; conduct of Annual Survey of Industries and Economic
Censuses, compilation of Index of Industrial Production, as well as
Consumer Price Indices. It also deals with various social statistics,
training, international cooperation, Industrial Classification etc.

The ASI extends to the entire country. It covers all factories registered
under particular sections of the Factories Act, 1948 i.e. those factories
employing 10 or more workers using power; and those employing 20 or
more workers without using power.

Q Source: Improvisation: Page 18: Standard Xth Economics NCERT:


Understanding Economic Development

80. South Korea lies between


a) Yellow Sea and Sea of Japan
b) Sea of Japan and Pacific Ocean
c) Korean Strait and Bungo channel
d) South China Sea and Pacific Ocean

Solution: a)
Learning: Answer cannot be South China Sea as it lies down South. So,
D is eliminated.


INSIGHTS PRELIMS TEST SERIES 2017

www.insightsonindia.com www.insightsias.com
TEST 4 Solutions

Bungo channel lies near Japan. So, C is also eliminated.

Map is as follows.

Q Source: East Asia: Map-based questions

81. Despite India being the largest producer of Pulses in the


World, India imports Pulses to meet domestic demand. Why India
cant produce enough to meet domestic demand?
1. Pulses are one-season crop and thus can be grown only in Rabi
season which severely reduces availability in other seasons.


INSIGHTS PRELIMS TEST SERIES 2017

www.insightsonindia.com www.insightsias.com
TEST 4 Solutions

2. Pulses require intensive irrigation facilities and heavy rainfall,


which are not fulfilled due to shortage of Monsoon rainfall and
irrigation bottlenecks.

Which of the above is/are correct?

a) 1 only
b) 2 only
c) Both 1 and 2
d) None

Solution: d)
Justification: The shortage of pulses and its spiralling prices has been
a major food security issue in India.

Statement 1: Pulses are grown in both Seasons Kharif and Rabi. So, 1
is wrong.

Statement 2: Pulses need less moisture and survive even in dry


conditions. So, 2 is clearly wrong.

Learning: Some of the reasons why domestic production in not enough.

Huge demand due to high population

Pulses are exported despite a shortfall in domestic production and


higher consumption.

Incentive structure is distorted against pulses by high government


subsidies for cereals (e.g. MSP).

Pulses are risky crops as per farmers. For e.g. in UP, Nilgai
destroys huge pulse crops within hours. Little protection is offered
by the local government.

Productivity of pulses is lower than world average, and just one-


third of that in US and Canada. Poor quality of seeds and input are

Q Source: Improvisation: Page 40: Xth Geography NCERT:


Contemporary India - II


INSIGHTS PRELIMS TEST SERIES 2017

www.insightsonindia.com www.insightsias.com
TEST 4 Solutions

82. Which of the following nutrients are absorbed by plants from


air?
1. Hydrogen
2. Sulphur
3. Boron
4. Carbon

Select the correct answer using the codes below.

a) 1, 2 and 4 only
b) 2 and 3 only
c) 1 and 4 only
d) 4 only

Solution: d)
Justification: Soil provides the following:

(i) Macronutrients: nitrogen, phosphorus, potassium, calcium,


magnesium, sulphur

(ii) Micronutrients: iron, manganese, boron, zinc, copper, molybdenum,


chlorine

Air provides carbon and oxygen only.

Water provides hydrogen and oxygen only. So, only 4 is the correct
statement.

Q Source: Page 206: 9th Science NCERT

83. Consider the following matches of cooperatives with what


they are associated with.
1. Krishak Bharati Cooperative Limited (KRIBHCO) : Fertilizers
2. Academy of Development Science (ADS): Maharashtra
3. Mother Dairy: National Dairy Development Board (NDDB)


INSIGHTS PRELIMS TEST SERIES 2017

www.insightsonindia.com www.insightsias.com
TEST 4 Solutions

Select the correct answer using the codes below.

a) 1 and 2 only
b) 1 and 3 only
c) 2 only
d) 1, 2 and 3

Solution: d)
Justification: Statement 1: It is an Indian cooperative society that
manufactures fertilizer, mainly urea. It is registered under the Multi-
State Cooperative Societies Act 2002. KRIBHCO is not owned nor
controlled nor financed by the Government of India / Any State
Governments

Statement 2: In Maharashtra, Academy of Development Science (ADS)


has facilitated a network of NGOs for setting up grain banks in different
regions. ADS organises training and capacity building programmes on
food security for NGOs. Grain Banks are now slowly taking shape in
different parts of Maharashtra.

Statement 3: Mother Dairy was commissioned in 1974 as a wholly


owned subsidiary of the National Dairy Development Board (NDDB). It
was an initiative under Operation Flood, the world's biggest dairy
development program launched to make India a milk sufficient nation.

Q Source: Page: Standard IXth Economics NCERT

84. Arrange these places in East Asia from North to South.


1. Pyongyang
2. Seoul
3. Tokyo

Select the correct answer using the codes below.

a) 123
b) 231


INSIGHTS PRELIMS TEST SERIES 2017

www.insightsonindia.com www.insightsias.com
TEST 4 Solutions

c) 132
d) 213

Solution: a)
Justification: Pyongyang is the capital of the Democratic People's
Republic of Korea (DPRK), commonly known as North Korea, of which it
is the largest city. DPRK was in news due to nuclear tests, hence the
coverage.

Seoul is the capital city of South Korea. Tokyo is the capital of Japan.

Q Source: East Asia: Map-based questions

85. Land degradation is major problem in India. Which of the


following factors is responsible for the degradation of the largest
land area?
a) Water Erosion
b) Salinity and Alkalinity
c) Wind Erosion
d) Forest degraded area

Solution: a)


INSIGHTS PRELIMS TEST SERIES 2017

www.insightsonindia.com www.insightsias.com
TEST 4 Solutions

Justification: The table covers major factors behind degradation.

Learning: Rajasthan is the state with maximum degraded land nearly 1


lakh sq. km. Union territories have the least around 100 sq. km..

Other states like MP, Maharashtra, AP, J&K have nearly 50-
70,000 sq. km. UP, HP, Gujarat Odisha, Jharkhand, Karnataka
have around 10-20,000 sq. km.

In states like Jharkhand, Chhattisgarh, Madhya Pradesh and


Orissa deforestation due to mining have caused severe land
degradation.

In states like Gujarat, Rajasthan, Madhya Pradesh and


Maharashtra overgrazing is one of the main reasons for land
degradation.

Q Source: Page 7: Xth Geography NCERT: Contemporary India - II


INSIGHTS PRELIMS TEST SERIES 2017

www.insightsonindia.com www.insightsias.com
TEST 4 Solutions

86. Consider the following about Self-Help groups (SHGs) in


India.
1. They are governed by the same Constitutional provisions that
regulate cooperatives.
2. A SHG can apply for credit support to niche banks like the
Bharatiya Mahila Bank.
3. SHGs must be compulsorily registered with the government.

Select the correct answer using the codes below.

a) 1 and 2 only
b) 1 and 3 only
c) 2 only
d) 1, 2 and 3 only

Solution: c)
Justification: Statement 1: Right to form Cooperatives is a
fundamental right and is governed by Constitutional provisions. The
same does not apply to SHG which is a small voluntary association.

Statement 2: BMB helps women get credit on easier terms than


commercial banks and also gives loan to SHGs and enterprising women.

Statement 3: A SHGs may or may NOT be registered. One person from


one family can become a member.

Q Source: Improvisation: Page 51: Standard Xth Economics NCERT:


Understanding Economic Development

87. Which of the following is NOT an example of a trade barrier?


a) Allowing only a fixed amount of commodities to be exported
or imported irrespective of supply-demand situations
b) Capping foreign direct investment in sensitive sectors
c) Imposing unreasonable standards on quality of imports
d) Tax on imports

Solution: b)


INSIGHTS PRELIMS TEST SERIES 2017

www.insightsonindia.com www.insightsias.com
TEST 4 Solutions

Justification: Option C: A barrier to trade is a government-imposed


restraint on the flow of international goods or services. The most
common barrier to trade is a tariffa tax on imports. Tariffs raise the
price of imported goods relative to domestic goods (goods produced at
home). So, D is correct.

Option A: An import quota is a type of protectionist trade restriction that


sets a physical limit on the quantity of a good that can be imported into a
country in a given period of time. So, A is correct.

Option B: FDI is related to the capital sector and overall investment


policy of the nation. FDI is not considered a trade component. So, B is
wrong.

Option C: Such standards can effectively clog imports from competitor


nations, for e.g. China in case of India.

Q Source: Improvisation: Page 64: Standard Xth Economics NCERT:


Understanding Economic Development

88. The Great Barrier Reef is the largest living thing on Earth
and even visible from outer space. It is located
a) At Soloman Islands
b) Off the coast of Queensland in Northeastern Australia
c) Near Timor Sea in Southern Australia
d) South of Kimberley Plateau

Solution: b)
Justification: The Great Barrier Reef is the world's largest coral reef
system composed of over 2,900 individual reefs and 900 islands
stretching for over 2,300 kilometres!


INSIGHTS PRELIMS TEST SERIES 2017

www.insightsonindia.com www.insightsias.com
TEST 4 Solutions

It supports a wide diversity of life and was selected as a World Heritage


Site in 1981. CNN labelled it one of the seven natural wonders of the
world.

Q Source: Adjunct questions: South-east Asia: Map-based questions

89. In India, Alluvial Soils can be found in


1. Eastern Coastal Plains
2. Western India
3. Southern India

Select the correct answer using the codes below.

a) 2 and 3 only
b) 1 only
c) 1 and 3 only
d) 1, 2 and 3

Solution: d)
Justification: Alluvial Soils are the most widely spread and important
soil. In fact, the entire northern plains are made of alluvial soil.

These have been deposited by three important Himalayan river systems


the Indus, the Ganga and the Brahmaputra.


INSIGHTS PRELIMS TEST SERIES 2017

www.insightsonindia.com www.insightsias.com
TEST 4 Solutions

These soils also extend in Rajasthan and Gujarat through a narrow


corridor. Hence, 2 is correct.

Alluvial soil is also found in the eastern coastal plains particularly in the
deltas of the Mahanadi, the Godavari, the Krishna and the Kaveri
(Southern India) rivers. Hence, 1 and 3 are correct.

Q Source: Page 8: Xth Geography NCERT: Contemporary India - II

90. As per the provisions of the Consumer Protection Act, 1986


1. Consumer Protection Councils are to be established by the
Central and State governments.
2. A Consumer Disputes Redressal Forum known as the "District
Forum" is to be established by the State Government in each
district.

Which of the above is/are correct?

a) 1 only
b) 2 only
c) Both 1 and 2
d) None

Solution: c)
Justification: Statement 1: The Central Council shall consist of the
following members, namely:

the Minister in charge of the consumer affairs in the Central


Government, who shall be its Chairman, and

such number of other official or non-official members


representing such interests as may be prescribed.

Statement 2: Each District Forum shall consist of,

a person who is, or has been, or is qualified to be a District Judge,


who shall be its President;


INSIGHTS PRELIMS TEST SERIES 2017

www.insightsonindia.com www.insightsias.com
TEST 4 Solutions

two other members, one of whom shall be a woman

Learning: Locally known as consumer forums or consumer protection


councils. They guide consumers on how to file cases in the consumer
court. On many occasions, they also represent individual consumers in
the consumer courts.

These voluntary organisations also receive financial support from the


government for creating awareness among the people.

Under COPRA, a three-tier quasijudicial machinery at the district, state


and national levels was set up for redressal of consumer disputes.

Q Source: Page 84: Standard Xth Economics NCERT: Understanding


Economic Development

91.Excessive use of Chemical fertilizers on crop fields may result in


1. Soil degradation in the long run
2. Pollution of local water bodies and groundwater
3. Reduction in native microorganism population of the farm in
the long run

Select the correct answer using the codes below.

a) 1 and 2 only
b) 2 and 3 only
c) 1 only
d) 1, 2 and 3

Solution: d)
Justification: Statement 1: For e.g. the consumption of chemical
fertilizers in Punjab is highest in the country. The continuous use of
chemical fertilizers has led to degradation of soil health.


INSIGHTS PRELIMS TEST SERIES 2017

www.insightsonindia.com www.insightsias.com
TEST 4 Solutions

Punjab farmers are now forced to use more and more chemical fertilizers
and other inputs to achieve the same production level. This means cost
of cultivation is rising very fast.

Statement 2: Some minerals dissolve in water and are immediately


available to plants. But others which are not readily absorbed may not be
retained in the soil for long. They may escape from the soil and pollute
groundwater, rivers and lakes.

Statement 3: Chemical fertilizers can also kill bacteria and other


microorganisms in the soil. This means some time after their use, the soil
will be less fertile than ever before.

Q Source: Page 6: Standard IXth Economics NCERT

92. The Mongoloid Shompen Tribes can be found in which of the


following protected areas in India?
a) Nokrek Biosphere Reserve
b) Agasthyamala Biosphere Reserve
c) Nilgiri Biosphere Reserve
d) None of the above

Solution: d)
Justification: The Shompen are as precariously poised on the brink of
extinction as the four other hunter-gatherer tribes of the Andaman and
Nicobar Islands. They are found in the Great Nicobar Reserve.

Classified as a Primitive Tribal Group (PTG) along with the four


other tribes, the Shompen are, unlike the other primitive tribes of
the Andaman Islands, not of negrito but of mongoloid stock.

Their light yellow-brown skins, straight hair, narrow eyes and


stocky build give them a strong resemblance to the people of
Myanmar and Indonesia.


INSIGHTS PRELIMS TEST SERIES 2017

www.insightsonindia.com www.insightsias.com
TEST 4 Solutions

Unlike the major islands of the Andamans and some Nicobar


Islands, Great Nicobar was, by and large, undisturbed by
incursions of outsiders until the late 1960s.

However, the major influx since 1969 has disturbed Shompen


populations.

Several development activities are currently happening in Great


Nicobar, all with an inevitable deleterious impact on the Shompen.
Some are security-related given the strategic location of Great
Nicobar almost at the southern end of India and its proximity to
many international shipping routes.

Q Source: Biosphere Reserves of India in the UNESCO Man and


Biosphere Network

93. Consider the following statements.


1. The Club of Rome was one of the earliest organizations to
vouch for sustainable resource conservation.
2. The recommendations of Brundtland Commission, 1987 were
published in the document Small is Beautiful.
3. The book Our Common Future, authored by Pandit Nehru,
was released at the first World Sustainable Development
Commission.

Select the correct answer using the codes below.

a) 1 and 2 only
b) 2 and 3 only
c) 1 only
d) None of the above

Solution: c)


INSIGHTS PRELIMS TEST SERIES 2017

www.insightsonindia.com www.insightsias.com
TEST 4 Solutions

Justification: Statement 1: At the international level, the Club of


Rome advocated resource conservation for the first time in a more
systematic way in 1968.

Statement 2 and 3: Subsequently, in 1974, Gandhian philosophy was


once again presented by Schumacher in his book Small is Beautiful.

The seminal contribution with respect to resource conservation at the


global level was made by the Brundtland Commission Report, 1987.

This report introduced the concept of Sustainable Development and


advocated it as a means for resource conservation, which was
subsequently published in a book entitled Our Common Future. So, 2
and 3 are wrong.

Another significant contribution was made at the Earth Summit at Rio


de Janeiro, Brazil in 1992.

Q Source: Page 5: Xth Geography NCERT: Contemporary India - II

94. In 2016, which of the following Indian sites were added to


the UNESCOs World Network of Biosphere Reserves?
1. Agasthyamala
2. Nanda Devi
3. Gulf of Mannar

Select the correct answer using the codes below.

a) 2 only
b) 1 and 3 only
c) 1 and 2 only
d) 1 only

Solution: d)
Justification: Statement 1: The ABR covers the Shendurney and
Peppara wildlife sanctuaries and parts of the Neyyar sanctuary in Kerala
and the Kalakad Mundanthurai Tiger Reserve of Tamil Nadu.


INSIGHTS PRELIMS TEST SERIES 2017

www.insightsonindia.com www.insightsias.com
TEST 4 Solutions

The area falls in the Malabar rainforests and is one of the noted hotspot
areas because of its position in the Western Ghats

Statement 2: Located in Uttarakhand, it was added in 2004.

Statement 3: It was added much before in 2001 considering its


importance in maintaining the marine ecosystem in Southern Indian
Ocean.

Q Source: Biosphere Reserves of India in the UNESCO Man and


Biosphere Network

95. Consider the following about Agenda 21.


1. It was adopted at the Rio Convention.
2. It distributes financial resources to G-20 + 1 countries to
mitigate climate change.
3. One major objective of the Agenda 21 is that every local
government should draw its own local Agenda 21.

Select the correct answer using the codes below.

a) 1 and 2 only
b) 2 only
c) 1 and 3 only
d) 1, 2 and 3

Solution: c)
Justification: It is the declaration signed by world leaders in 1992 at
the United Nations Conference on Environment and Development
(UNCED), which took place at Rio de Janeiro, Brazil.

It aims at achieving global sustainable development. It is an agenda to


combat environmental damage, poverty, disease through global co-


INSIGHTS PRELIMS TEST SERIES 2017

www.insightsonindia.com www.insightsias.com
TEST 4 Solutions

operation on common interests, mutual needs and shared


responsibilities.

One major objective of the Agenda 21 is that every local government


should draw its own local Agenda 21, i.e. implement the 21 points at the
local level as they are implemented at the international and national
level.

Q Source: Page 3: Xth Geography NCERT: Contemporary India - II

96. Sanitary and phytosanitary measures of WTO are related to


a) Health and safety regulations
b) Protection to biodiversity
c) Providing funding support to member nations to achieve zero
defection
d) Both (a) and (b)

Solution: d)
Justification: The Agreement on the Application of Sanitary and
Phytosanitary Measures, also known as the SPS Agreement, is an
international treaty of the World Trade Organization.

Under the SPS agreement, the WTO sets constraints on member-states'


policies relating to food safety (bacterial contaminants, pesticides,
inspection and labelling) as well as animal and plant health
(phytosanitation) with respect to imported pests and diseases.

For e.g. exotic species can wipe out native population if not handled
properly. Non-quarantined containers can bring disease to a nation.

SPS ensures that unrestricted imports do not adversely affect the health
and safety of trading nations.


INSIGHTS PRELIMS TEST SERIES 2017

www.insightsonindia.com www.insightsias.com
TEST 4 Solutions

Q Source: WTO website http://www.wto.org: Improvisation: Page 54:


Standard Xth Economics NCERT: Understanding Economic
Development

97. Which of the following statements about Regur Soils in


INCORRECT?
a) It has the highest porosity amongst all soil types.
b) It is generally poor in phosphoric contents.
c) They are suitable for growing Cotton.
d) It can be found in the Deccan trap and Malwa Plateau.

Solution: a)
Justification: Black soil is also known as regur soils.

Black soil is ideal for growing cotton and is also known as black cotton
soil.

Option A: The black soils are made up of extremely fine i.e. clayey
material. They are well-known for their capacity to hold moisture.

They develop deep cracks during hot weather, which helps in the proper
aeration of the soil.

Option B: They are rich in soil nutrients, such as calcium carbonate,


magnesium, potash and lime. These soils are generally poor in
phosphoric contents.

Option D: It is believed that climatic condition, along with the parent


rock material, are the important factors for the formation of black soil.

This type of soil is typical of the Deccan trap (Basalt) region spread
over northwest Deccan plateau and is made up of lava flows.


INSIGHTS PRELIMS TEST SERIES 2017

www.insightsonindia.com www.insightsias.com
TEST 4 Solutions

They cover the plateaus of Maharashtra, Saurashtra, Malwa,


Madhya Pradesh and Chhattisgarh and extend in the south east
direction along the Godavari and the Krishna valleys.

Q Source: Page 9-10: Xth Geography NCERT: Contemporary India - II

98. Consider the following about Manganese Nodules found in


the Ocean.
1. They are generally found within shells floating at the sea surface
along with phytoplankton.
2. They mainly contain manganese, cobalt, nickel and copper.
3. Crude oil is extracted from these nodules in the Indian Ocean.

Select the correct answer using the codes below.

a) 2 only
b) 1 only
c) 1 and 3 only
d) 2 and 3 only

Solution: a)
Justification: Statement 1 and 2: Polymetallic nodules (manganese
nodules) are rock concretions on the sea bottom formed of concentric
layers of iron and manganese hydroxides around a core. It mainly
contains manganese, cobalt, nickel and copper.

Statement 3: They do not contain crude oil.

India has got the right to mine manganese nodules from the bed of the
Indian Ocean from that area which lies beyond the exclusive economic
zone.

Learning: Nodule mining could affect tens of thousands of square


kilometers of deep sea ecosystems. Nodule regrowth takes decades to
millions of years and that would make such mining an unsustainable and
nonrenewable practice. Thus, nodule mining could cause habitat


INSIGHTS PRELIMS TEST SERIES 2017

www.insightsonindia.com www.insightsias.com
TEST 4 Solutions

alteration, direct mortality of benthic creatures (living near sea bed), or


suspension of sediment, which can smother filter feeders (suspension
feeding animals).

Q Source: Glossary: Xth Geography NCERT: Contemporary India - II

99. The inscriptions of the Pallavas mention a number of local


assemblies including the Ur. Consider the following about it.
1. The assembly was found in areas where the land owners were
generally Brahmins.
2. Agriculture was not practiced in regions administered by the
Ur.

Which of the above is/are correct?

a) 1 only
b) 2 only
c) Both 1 and 2
d) None

Solution: d)
Justification: The assemblies mentioned in Pallava inscriptions
include the sabha, which was an assembly of brahmin land owners.

This assembly functioned through subcommittees, which looked after


irrigation, agricultural operations, making roads, local temples, etc.

The ur was a village assembly found in areas where the land owners were
not brahmins. And the nagaram was an organisation of merchants.

Q Source: Revision past Test syllabus


INSIGHTS PRELIMS TEST SERIES 2017

www.insightsonindia.com www.insightsias.com
TEST 4 Solutions

100. What is common between the birds Stork, Flamingo, Pintail


Duck and Curlew?
a) They have been depicted in Sanchi inscriptions.
b) Their habitat ranges are extremely narrow due to which they
are greatly threatened by climate change and habitat
destruction.
c) They have flat topped bodies that allow them to survive in
hot regions.
d) They are migratory birds.

Solution: d)
Justification: Some birds migrate to our country in the winter season
every year such as Pelican, Siberian Crane, Stork, Flamingo, Pintail
Duck, Curlew.

Siberian Cranes migrate from Siberia; they arrive in December and stay
till early march.

Q Source: Revision past Test syllabus


INSIGHTS PRELIMS TEST SERIES 2017

Вам также может понравиться